Dha,prometric Questions

  • Uploaded by: reginald reginald
  • 0
  • 0
  • February 2021
  • PDF

This document was uploaded by user and they confirmed that they have the permission to share it. If you are author or own the copyright of this book, please report to us by using this DMCA report form. Report DMCA


Overview

Download & View Dha,prometric Questions as PDF for free.

More details

  • Words: 32,517
  • Pages: 89
Loading documents preview...
Diya Coaching Centre for Nurses One candle wipes out darkness…….

1.

The nurse administrated a dose of morphine sulphate as prescribed to a patent who is in PACU. The patient appears to be resting comfortably the respiratory rate is 8 and O2 saturation is 86% via O2 cannula. The Nurse should immediately administer A. Flumazenil(Romazicon) B. Midazolum (versed) C. Nalaxone D. Ondansetron 2. A patient schedule for a major surgery in one hour is very nervous and upset. Which of the following medications would the nurse administrator to relax this patient? A. Meperidine hydrochloride (Demerol) B. Scopolumine (Tranderm-scop) C. Pentobarbital sodium (Nembutal sodium) D. Trazodone Hydrochloride (Trazdone) 3. A patient with poor wound healing and poor appetite has an order to begin total parentral nutrition(TPN). Waiting for the TPN solution to arrive from the pharmacy the Nurse should obtain A. A pair of sterile glove B. An infusion Pump C. IV tubing with amicro-drip chamber D Providone- iodine (Betadine) swabs. 4. A nurse is caring for a parent receiving TPN The patient reports the sudden onset of feeling short of breath and anxious. The nurse hears crackles in bilateral lower lobes of the lungs and the patients O2 saturation is 90% on room air. The nurse nest immediately. A. Turn off the TPN B. Notify the physician C Assess the patient’s capillary blood glucose level D. Attempt to suction the patient’s airway. 5. A nurse just started total parental nutrition as prescribed for a patient with severe dysphagia, low pre albumin levels. In one to hours the nurse should .................. assessing the patient. A. Blood glucose level B. Weight C. Liver D O2 saturation 6. The nurse is planning care for several children who were admitted during the shift. Daily weight should be of the plan of care for the child who is receiving A. TPN B. Supplement O2 C. Intravenous anti-ineffective D. Chest physiotherapy 7. The nurse is caring for a 4 year old.... a diagnosis of cystic fibrosis and pneumonia. The child is feeling better on the 3rd day of the hospitalized and “wants to play”. What would be the best choice of entertainment? A. Blowing bubbles Compiled by : V.Devi | 9176868900

Page 1

Diya Coaching Centre for Nurses One candle wipes out darkness…….

B. Looking at picture books. C. Watching videos D. Riding in a wagon 8. A nurse in caring for an 8 year old male with cystic fibrosis, based on the disease condition what nursing intervention should the nurse expect to perform? A Restrict sodium and fluid intake B.Give anti-diarrheal medication C. Discourage coughing after postural drainage pancreatic enzy D.Administrater pancreatic enzymes with each meal 9. A nurse is caring for a child with a diagnosis of cystic fibrosis and pneumonia. The plan of care includes Nebulizer Rx and chest physiotherapy. The nurse should perform chest physiotherapy A. Continusly during nebulizer Rx B. Prior to the nebulizer Rx C. After the Nebulizer Rx. D. Intermittently during nebulizer Rx 10. When conducting discharge teaching for the parent of child newly diagnosed with cystic fibrosis. Which of the following statement by the parent indicates the need for further teaching? A. weekly weights help evaluate effectiveness of nutritional intervention B. Weekly weights help the doctor know if my child is absorbing nutrients C. Weekly weights re-assure that my child recovery is progressing D. Weekly weights help the doctor know if my child needs additional enzymes 11. While caring for a pt with an ileostomy, the nurse would expect the ostomy to be located in which quadrant of abdomen? A. Right lower B. Left lower C. Left Upper D. Right Upper 12. A patient has been assessed and found to have a severe dysphagia and will need long term nutritional support. Which one of the following types of feeding would most likely to be beneficial for the patient? A. Gastrostomy B. Parenteral C. Nasogastric D. Nasoduodenal 13. A Surgeon instructs a nurse to serve as a witness to an elderly patient informed consent for surgery. During explanations to the pt it becomes clear and the patient is confused and does not understand the procedure but reluctantly signs the consent form. The Nurse should A. Sign the form as a witness making a notion that pt did not appear to understand B. Not sign the form as a witness and notify the nurse supervisor C. Not sign the form and tell surgeon that the pt dosen’t understand the procedure. D. Sign the form and tell surgeon that the pt dosen’t understand the procedure.

Compiled by : V.Devi | 9176868900

Page 2

Diya Coaching Centre for Nurses One candle wipes out darkness…….

14. The nurse evaluating the pt with an end stage COPD. The pt has not achieved any of the goals in the plan of care. The spouse reports concerns about the pt’s mood and increased dependency. What action should the nurse take first? A. Continue care plan for one more month B. Refer pt to psychiatric services C. Collaborate with pt and spouse to revise the care plan D. Revise the care plan based on spouses input 15. A home care pt with COPD reports an upset stomach. The Pt is taking ....................................................................................................................................... A. Theo-dur at empty stomach B. Theo-dur and azmacort at the same time C Theodur and azmacort 12 hour apart D. Theodur at night 16. When giving post operative discharge instructions to a patient who had abdominal surgery all of the following regarding would healing are true except? A.Bathing to soak abdomen is preffered over showing B. Avoid tight belts and cloths which seems that may rub the wound C.Pain medication may effect ability to drive D. Irregular bowel movements can be expected 17. A nurse giving post operative discharge instruction to a patient who had abdominal surgery. When teaching the patient about wound healing all of the following are true except? A.Wound may feel tightly or itching as healing occurs B. Scabs promote infection of the new skin underneath them C. Numbmness or a slight pulling sensation is normal D. Wound should not have any drainage 18. A 12 year old child who has been diagnosed with insulin dependent mellitus(IDDM) since age 3 comes to the clinic for a routine visit. The patient has begin to self manage care with parental supervision. The patient injects 28 units of NPH insulin every morning and 8 units at bed time. The patient checks blood sugar 4 times every day The patient’s weight is stable and diet is unchanged. However the patient report several hypoglycaemic reactions every week. The nurse knows the most weekly cause is that:A. The patient is not eating the adequate number of calories reported B. The dosage of insulin may need to be decresed as the patient continues grow C.There may be changes in exercise, stress level or the beginning of growth spurt D. The patient may not be competent in techniques of drawing up and injection insulin. 19. A nurse visits a patient at home who dosent understand how to take a new prescribed medication. The prescription reads 5ml PO tds P.C meals The nurse explains to the patient that the correct way to take the medication is; A. 1 table spoon by mouth3 times a day before meals B.1 teaspoon by mouth 3 times a day after meals C.1 teaspoon by mouth 3 times a day before meals D. 1 tablespoon by mouth 3 times a day after meals Compiled by : V.Devi | 9176868900

Page 3

Diya Coaching Centre for Nurses One candle wipes out darkness…….

20. The nurse is caring for a patient who had major abdominal surgery under GA hours ago. An appropriate goal for the patient includes A. Having minimal fine crackles in the base of lungs .............................................................. ......................................................................... 21. While caring for a child with a ventriculo peritoneal shunt revision the nurse finds the patient lying with the head and feet flexed back. The nurse should call for help and prepare for A. Spinal tap B. Shunt culture C. Electro cardiogram D.Ventricular tap 22. A patient undergoing treatment for cancer with bone metastasis is experiencing severe pain. Which of the following treatment would the nurse most likely expect to improve the patient’s pain control? A Adjuvant radiation therapy B. Palliative radiation therapy C. Curative radiation therapy D. Ventricular Tap 23. During surgery requiring general anesthesia the patient’s heart stops and carotid pulse is not palpated How many compressions per minutes should be administrated? A.50 B.60 C.80 D.100 24. When teaching a community class on cerebrovascular accidents, which of the following should partiapants of the class know at the completion of the class? A. Muscle and ligament damage is not reversible B. Expressive aphasia is resolved by voice rest C. There is a risk for mood disorders such as depression D. Liquids should be consumed at the same time as solids foods 25. A community health care nurse visits a pt who had a CVA. The pt is at risk for deficient volume due to voluntary reduction fluid intake to avoid the use of bathroom The Nurse educates the pt on the importance of drinking fluids and maintaining hydration. Which indicates efficient nursing intervention? A. Amber color urine B. Respiration of 35 C. Tachycardia D. Moist mucous membrane 26. A home health nurse is visiting a pt following a CVA The pt is having trouble sleeping and is feeling sad. The pt’s spouse tells the nurse that the patient is not eating much and often cries when no one is watching. Which of the following would be the nurse’s most likely intervention? A. Assess for changes in congnitive abilities B. Complete a depression index Compiled by : V.Devi | 9176868900

Page 4

Diya Coaching Centre for Nurses One candle wipes out darkness…….

C. Strengthen family coping methods D. Screen for pain 27. A home health Nurse is visiting a pt who recently suffered a CVA the nurse would most likely implement which of the following intervention to prevent muscle and ligament deformities? A. Daily moist heat and isometric exercises B. Daily balance training and routine medications for pain C. Instruct the pt to use non-affected side to perform activities of daily living D. Daily range of motion exercises 28. A nurse assigned to do a home visit for an 81 yr old pt. The pt lives at home with an adult care taken and is completely bed-bound following a CVA, 2weeks ago. In planning care giver education, the nurse should be prepared to instruct the care taker in A. How to select Nursing home for pt B. Performing passive range of motion exercises C. The importance of avoiding viscous drinks D. Forming a local chapter of care giver support group 29. A home care nurse makes a follow-up visit to a pt who recently suffered a CVA. The pt is mobile and able to perform activities of daily living. However the pt has not sleeping and has lost weight due to lack of appetite. The pt also feels overwhelmed with sadness Which of the following is the most appropriate evaluation A. Pt’s progress is as expected and no further intervention is necessary B. Pt needs referral to nutritionist C. Pt needs intervention for depression D. Pt needs sleeping medication 30. A patient admitted with CVA is unable to chew or swallow food with risk for aspiration. The nurse would anticipate receiving which of the following order for the patient? A. Give no food by mouth and start IV hydration B. Start a pureed diet with thickened liquids C. Refer the patient to psychiatrist for depression related to the CVA D. Refer the patient to physical therapy for muscle strengthening 31. While the nurse is administering a large volume enema, the patient complaints of cramps the nurse should A. increase the flow rate B. Lower the fluid container C. Elevate the head of bed D. Gently message the abdomen 32. A home health nurse has entered a home to complete an admission assessment on a pt who has a methicillin resistant staphylococcus aureus (MRSA) UTI. The pt will receive IV anti-infective via a peripherally inserted central catheter for 3 weeks. Which of the following actions should the nurse take first? A. Shake the patient’s hand B. Place the nursing supply bag on a clean dry surface C. Obtain pt’s written consent for home health care Compiled by : V.Devi | 9176868900

Page 5

Diya Coaching Centre for Nurses One candle wipes out darkness…….

D. Perform hand hygiene per the agency protocol. 33. A home health nurse is teaching a family member about the care of pt’s peripherally inserted central catheter(PICC).Which of the following would be appropriate for the nurse to make? A. Place the used IV tubing in a leak proof container. B. You will need to put on a disposable face mask before you connect the intraveinous tubing to the port of the PICC. C. The port of PIC catheter will need to be cleansed the povidone-iodine(Betadine) after the infusion is completed. D. The empty medication container can be place in the same container as your household refuses. 34. A pt had a craniotomy with resection of a non-malignant neoplasm for the temporal lobe. The pt’s vital signs are within the baseline normal range. The nurse observes that the pt has developed bilateral periorbital edema. Which of the following actions would be appropriate for the nurse to take? A. Apply cold compress to the pt’s eye. B. Apply warm compress to the pt’s eye. C. Elevate the head of the pt’s bed to 60 degree. D. Elevate the head of the pt’s bed to 45 degree. 35. To decrease the incidence of aspiration of gastric contents in a child hospitalized with severe burns, the nurse should position the head. A. Flat expect during meals. B.Elevate 30-45 degree during meals. C.Elevate 15-30 degree for 12 hours after meals. D.Elevate 45 degree all times. 36. A home health nurse visit a patient with diabetes and primary open angle glaucoma. The patient take metformin (Glyciphage) 500 mg OD for diabetes &timolol ophthalmic solution BD in each eye for glaucoma. Which of the following elevations indicate that the patient is compliant with glaucoma management?. A. Patient has not been taking glyciphage. B. Patient has tearing of the eye. C. Patient has not refilled prescription for timolol in 3 months. D. Patient has yellow discharge from the eyes. 37. A pt is having difficulty with cognitive abilities after a stroke. What part of brain is affected? A. Mid brain B. Cerebrum C. Medulla oblongata D. Cerebellum 38. A 16 year old patient present to the clinic requesting birth control; with the diagnosis of health seeking behaviour the best goals have the pt: A. Verbalising , understanding of safe sex production and following safe sexual practices in all encounters.

Compiled by : V.Devi | 9176868900

Page 6

Diya Coaching Centre for Nurses One candle wipes out darkness…….

B Not engaging in sexual encounters until she is 18 yrs of old and maintaining a healthy life style. C. Recognizing the sign of pregnancy, the symptom of STD D. Understanding safe sexual practice and use of condom to prevent pregnancy and STD 39. A nurse plan to teach a group of 20-25 years old women about oral contraceptives. The nurse should instruct that oral contraceptives may; A. Increase the risk of pelvic inflammatory disease B. Cause acne to worsen C. Decrease the risk of breast and cervical cancer D. Decrease the risk of endometriosis 40. Following lumbar surgery a pt has a 4 mm surgical incision The incision is clean and the edges are well appropriate. The type of tissue healing is classified as which of the following? A. Primary intention B. Secondary intention C. Tertiary intention D. Superficial epidermal 41. A shrinkage device is applied after surgery for amputation of the leg. The goal of shrinking device is to form the residual limb into what shape? A. Cone B. Oval C. Mushroom D Cylinder c blunt end 42. A pt pulmonary embolus and a nurse diagnosis of impared gas exchange has an order to obtain ABG. The first intervention by the nurse is to; A. Perform Allen test B. Explain the procedure C. Gather the equipment D. Document the procedure 43. A pt is diagnosed with pulmonary hypertension. Which of the following nursing diagnosis should be the priority? A. Impaired gas exchange related to altered blood flow secondary to pulmonary capillary constriction B. Fatigue related to hypoxia C. Anxiety related to illness and loss of control D. Activity intolerance related to imbalance between O2 supply and demand related to right and left ventricular failure 44. A pt who had abdominal surgery in PACU which of the following nursing diagnosis take priority? A. Disturbed sleep pattern B Acute pain C. Risk for infection D. Infective airway clearence Compiled by : V.Devi | 9176868900

Page 7

Diya Coaching Centre for Nurses One candle wipes out darkness…….

45. While caring for a pt in the PACU a nurse observes the onset of rapid breathing and cyanosis, and narrowing blood pressure. The nurse’s should plan to A. Administer bolus glucose B. Suction airway C. Turn the pt to right side D. Administer IV fluids 46 . While caring for a pt in the PACU , who has developed hypovolemic shock, a nurse should position the pt. A. Flat with legs elevated B. In trendelenberg position C. With the head of bed elevated D. Completely flat 47. A pt vitrectomy and is about to be transported to the post anesthesia care unit(PACU) The pt should be placed in which of the following positions before transport to PACU? A Semi fowler’s B. prone C Dorsal recumbent D. Sims 48. While caring for a pt in PACU the nurse plan to keep pt warm. What is the important reason for the action is? A. To preserve nutritional stores B. to prevent cutaneous vessel dilation C. To decrease pt anxiety D. to lower risk of infection resulting from chilling 49 . A pt had a total abdominal hysterectomy 2 days-ago and has not been out of bed yet. The pt complaining left leg pain and swelling. What should the nurse do first? A. Gently massage the pts leg B. Assess the pt for Homan’s sign C. Assess the pt to reflex the left knee and lip D. Instruct the pt to reflex the left knee lip 50. To minimize a toddler from scratching and picking at a healing skin graft site, the nurse should utilize A. Mild sedative B. hand mittens C. Punishment D Distraction 51. The nurse teaching the mother of a 3 months old infant about bottle feeding. Which statement indicates the mother understands of appropriate procedure? A. It is Ok to prop the bottle on a pillow B. I can deed my whole milk C. I should hold my baby in slightly reclined position , close to my body D. I should warm the bottles in the microwave if they come out of the refrigeration Compiled by : V.Devi | 9176868900

Page 8

Diya Coaching Centre for Nurses One candle wipes out darkness…….

52. A 9 month old child who has had four ear infections in the past 6 months is being discharged which statement by the parent indicates the need for further discharge teaching? A. I should never put my baby to bed with bottle B. My child should not use a pacifier ago 6 months C. My child should drinks his bottle while laying flat in my lap D. My child should not be around people who smoke 53. A patient complaint of severe menstrual cramping, bleeding is not unusually heavy. And the patient has no uterine disorders. Which of the following interventions should the nurse anticipate the doctor will order to promote comfort? A. Acetaminophen B. Strict bed rest C. Heating pad to the back of neck D. Ibuprofen (motrin) 54. During pre operative preparation of a patient for amputation of the left leg The nurse has primary responsibility for A. witnessing the patient signature on the consent form B Explaining the procedure to the patient C.Explaining the risk of the surgery to the patient D.Making appropriate incision lines on the leg 55. A 52 years old is admitted to the nursing unit from the physician office with a diagnosis of acute cholecystitis. Physician order on admission include monitor vital signs for every 4 hrs. IV of ringers lactate 125ml/hr. 1500 calorie, low fat liquid diet, morphine sulphate 2 mg IV every2hours as need for pain, notify physician for sudden increase in frequency or intensity of pain, promethazine 12.5mg IV every 4 hours as needed for nausea or vomiting. Which of them following should the nurse plan to do FIRST? A. Remove any high foods from the patients room B. notify the dietitian of the diet order C. Obtain venous access and start Ringer’s lactate infusion D Obtain an emisis basin and clean linen from the bed side. 56. A patient brings a 10 month old infant into the department saying “my baby put a button in her mouth and now she is not breathing. After the nurse determines the infant is not breathing. What should the nurse do NEXT? A. Perform the Heimlich maneuver B. Initiate cardio pulmonary resuscitation (CPR) C. Administer 4 back blows D. Admistrater 4 thrusts midline on the patient back 57. An infant arrives in the emergency department not breathing and does have a pulse When starting cardiopulmonary resuscitation ,where is the correct place to assess for a pulse in this patient? A. Carotid B. Radial C.Brachial Compiled by : V.Devi | 9176868900

Page 9

Diya Coaching Centre for Nurses One candle wipes out darkness…….

D. Temporal 58. A 5 year old patient who underwent abdominal surgery suffers from deficient fluid volume related to nothing by month (NPO) status. Intravenous fluid therapy is given for hydration Which of the following indicates that the treatment is effective? A. Urinary output of 15ml/hr B. Respiratory rate 35 C. Heart rate 100 D. Good skin turgor 59. A 7 year child is brought to the physician’s office due to sudden onset of bright redness on the checks. The nurse observe that the child has a temperature of 380 c(1040F) with this the nurse suspect that the most like diagnosis would be A. Fifthe disease B. Rota virus C. Roscola Infantum D. Monitor patient for shock 60. What are the common ventilator complication? A. Hypotension B. Hyperventilation C. Bucking with Ventilator D. VAP 61. Expain the first management for digoxin toxicity? A. Administer digibind B.With hold the drug C. Administer beta blockers D. Monitor ECG 62. A patient came with burn on chest& arm which artery can be used for ABG? A. Radial B. Brachial C. Femoral D. Carotid 63. What is barrel chest? A. AP=transverse diameter B AP>TD C.APTD> 64. What is the care of a patient with diverticulosis? A.Provide high residue diet B. Provide low residue diet C.provide high residue diet without seed D. Low calorie diet 65. What all are the instruiction given to a patient using cane? Compiled by : V.Devi | 9176868900

Page 10

Diya Coaching Centre for Nurses One candle wipes out darkness…….

A Palce the cane close to the body B. Place the cane on the affected side C. Place the cane on the unaffected side D.Hold the cane with the hand in straight manner 66. Why gluteal muscle is not used in infants for IM injection? A. It have high vascularity B. It is immature C. High chance for muscle atrophy D. High chance for abscess formation 67 . Patient receiving penicillin suddenly develops red rashes. First action? A. Administer glucocorticoids B. Administer adrenaline C. Withhold the drug D. Administer antihistamines. 68. Complication of morphin sulphate? A. Respiratory suppression B. Hypersensitivity C. Vomiting D. Constipation 69. Iron rich food? A. organ meats B. green leafy vegetables C. citrus fruits D. Canned foods 70. Home care nurse conducting a health programme for screening lifestyle diseases by checking BP and blood glucose. This type of prevention is called......................... A. Primary prevention B. Secondary prevention C. Tertiary prevention D. Primodial prevention 71. A patient with severe depression is on anti-depressants after taking the drug the patient shows restlessness. What would be the reason? A. As a side effect B. Normal response C. Therapeutic response D. Adverse drug reaction 72. What are the complication of blood transfusion? A. Transfusion reaction B. Arrhythmias C. Hemorrhage D. Renal caliculi formation

Compiled by : V.Devi | 9176868900

Page 11

Diya Coaching Centre for Nurses One candle wipes out darkness…….

73. Among the following which is the complication of renal surgery? A. Renal failure B. Wound dehiscence C. Wound infection D. Hemorrhage 74. List down the steps of nursing process in order A. Assessment, nursing diagnosis, goal, intervention, implementation, evaluation. B. Nursing diagnosis, goal, intervention, assessment, implementation, evaluation. C. Intervention, implementation, evaluation, assessment, nursing diagnosis, goal. D. Assessment, nursing diagnosis, goal, implementation, intervention, evaluation. 75. List the foods that are rich in vitamin C? A. fresh fruits and vegetables. B.organ meats C. pulses D. wheat & wheat products. 76. Type of stool present in transverse colon? A. Semi-solid B. Solid C. Liquid D. Mesh 77. After restoration od ear, what is expected outcome after 5 days? A. Decreased the hearing B. Normal hearing C. Increased hearing D. None of the above 78. Medical management of parkinson’s disease? A. Levodopa B. Methyldopa C. Antibiotics D. None of the above 79. What is the antidote of pethedine? A. Flumazenil B. Diazepam C. Promethazine D. Nalaxone 80. What is the drug of choice for Rheumatic fever? A. Benzyl pencilline B. Benzathene pencilline C. D-Pencillamine D. Nistatin 81. A Women has pain in the iliac region and feel tenderness on palpation. Which diagnostic measure the doctor will order now? A.X-ray Compiled by : V.Devi | 9176868900

Page 12

Diya Coaching Centre for Nurses One candle wipes out darkness…….

B. USG C. CT D. Blood test 82. A patient after mastectomy should do which all types of exercises after 7 days of surgery? A. Aerobic exercise B. hand wall climbling exercise C. Quadriceps sitting exercise D. Gluteal setting exercise 83. A patient with mastoidectomy having perorbital edema. The nurse should do which intervention first? A. Lower the head end of bed B. Check BP C. Head end elevation D. Foot end elevation 84. A child after surgery for tetrology of fallot at home develops breathing difficulty. Which position the child should be placed? A. prone B. knee chest C.Supine D. Sims 85. The patient who is taking spirinolactone the nurse should remain which intervention with the patient? A. Limit K intake B. Take medicine at bed time C. Limit Na intake D. Limit calcium intake 86. A patient with 80% burns admitted to hospital. What is the priority nursing diagnosis? A. Impared body image B. Self care deficit C. Fluid Volume deficit D. Risk for infection 87. A G2P1, mother with pre-normal delivery came to hospital at 20 week of gestation. USG shows normal gestational age. For the second visit when she came at 28 week, her abdominal height shows 32 week gestation. What would be the reason? A. Any uterine fibroid B polyhydraminos C. Wrong gestational age D. Oligohydraminosis 88. An 18 month old baby after surgery received in PACU. Baby is spontaneously opening eye, an independent movements of extremities seen. (what is baby’s GCS) A. 15 B.10 C. 8 Compiled by : V.Devi | 9176868900

Page 13

Diya Coaching Centre for Nurses One candle wipes out darkness…….

D. 12 89. Immediately after delivery baby with apgar score is 8 and after 5 minutes the score is 10 which would be the next measuring prority? A.O2 administration B. prevent heat loss C. Suctioning D. Prevent infection 90. A patient in found to be unresponsive, at what rate CPR to be started? A.100 B.>100 C.<100 D. Atleast 100 91. Adverse effect of clozaril (Anti-psychotic) A. WBC B. RBC C. LFT D. Platelet count 93. Which one is dorsalis pedis artery? a. A b.B c.C d.D 94. Which one right median nerve? aA b.B c.C d.D 95. Not risk factor of Angina pectoris? A. Coronary artery disease B DM C. Smoking D. Decreased total cholesterol level 96. A 10 year old boy C/o fever since last 3 days. Along with dizziness and small rashes all the body. There are some small red spots with bluish white centre appeared on his buccal mucosa for last 2 days A. Mumps B. Diphtheria C Measles D. Chicken pox

Compiled by : V.Devi | 9176868900

Page 14

Diya Coaching Centre for Nurses One candle wipes out darkness…….

97. 10 year old osteogenic carcinoma boy admitted for chemotherapy and radiation. In which stage is his cancer A. I B.II C.III D.IV 98. Tricuspid beat exactly heared on A. 2nd Rt ICS B. Apex of heart C. 4th LTICS D. 3rd Lt ICS 99. Multiple sclerosis assessment findings except one is; A Abnormal reflexes B. Spasticity of lower extrermities C. Normal bladder function D. Confusion and euphorea 100. A 10 years old C/o eye irritation on Lt eye during visual examination doctor find that small scratches on left eye. But right eye is normal. The Prescribe antibiotics eye drops it should install in A. Left eye in lateral canthus B. On the left eye with supine portion C Both eyes D. On the left eyes with lawless position 101. How can you avoid insulin lipodystropy? A. Discontinue insulin B. reduce the dose C. Rotate the site D. Massage the site after each injection 102. In which part use can suspect solid fecal material A.Transverse colon B. Ascending colon C. Sigmoid colon D. Ileum 103. A patient’s ICD is accidently fell down during position change. What’s the nurse’s initial action? A. Call the physician B. Re-insert the tube C Sterile petroleum soaked guaze dressing on the site D Keep the tube in bedside bottle. 104. How can a nurse reduce infection in the CVP site? A. Dressing periodically. B. Proper hand washing C. Sterile dressing Compiled by : V.Devi | 9176868900

Page 15

Diya Coaching Centre for Nurses One candle wipes out darkness…….

D. Restrict the visitors 105. MRSA precautions? A. Reverse isolation B. Contact isolation C. Droplet isolation D. Airborne isolation 106. Antidot for calmpose? (diazepam) A. Flumazenil B Diazepam C. promethazine D. Nalaxone 107. A 91 days boy C/o lethargy and tiredness for last 2 days. He came to the clinic and nurse noticed that (picture view> tongue >under bluish spot with white center) What signs and symptoms.(koplicks spot)

A. Neuritis B Myocarditis C. Measles D Rubella 107. 70 year old lady complaints of fatigue and breathing difficulty during walking. Her vitals are normal. Nurse notify that her tongue is swollen red beefy colour. What is the best advice? A Iron administration B. Blood transfusion C. B12 administration D. Folic acid 108. A 11 year old girl was playing in the garden suddenly developed rashes on her hand and face. Mother bought her to the hospital. Nurse noticed that girl’s lips are swollen and tender. What is her initial action? A. Epinephrine S/c B. O2 administration C. Pain medication D. IVF 109. What is the following process Picture

A. Osmosis B. Diffusion C. Ultra filtration D. Hydrostatic pressure 110. A wound picture is given(wound in 5 cm, pink and granulating yellow shield present) Compiled by : V.Devi | 9176868900

Page 16

Diya Coaching Centre for Nurses One candle wipes out darkness…….

What type of wound is this? A. Arterial. B. Venous C. Criminal D. Negligence 112. A patient has 3rd stage of decubitus ulcer. The wound is non exudates and non proliferate what is the treatment of choice? A. Hydrocolloidal B. Hydrogel C. Adhesive transparent tape D. Sterile guaze 113. A nurse instructs a community education class on breast health. Which statement indicates understanding of the appropriate age to starts screening mammogram? A. At menopause B. At 65- years old C At the cessation of breast feeding D. At 40 years old 114. A patient is taught how to perform a breast self examination by a nurse. Which statement is best described the understanding of the proper procedure for doing a breast self exam? A Use the palm of the hand to feel for lumps B. Apply three different levels of pressure to feels breasts tissue. C. Stand when performing a breast self exam D. Perform self exam annually 115. When carrying for a child with ineffective airway clearance related to increased mucus production the nurse should encourage fluids to A. Maintain nutrition B Prevent boredom C. Stimulate coughing D. Thin secretions 116. A 59 year old patient with lung cancer and metastases to the bone is in the hospital for pain 10 in number scale(10 point scale) The best goal for this nursing diagnosis is A. Show no objects signs of pain B. Not complain of pain C. State pain is at tolerable level D. State that all pain is relieved 117. A patient with advanced skin cancer exhibiting cyanosis and edema of head and upper extremities. Which of the following intervention is most likely provide an immediate benefit for this patient? A. Place in a trendelenburg position. B. Position on the right side C. Elevate the head of the body D. Elevate extremeities 118. If a patient developes a complication during blood transfusion, the nurse first should be to do? Compiled by : V.Devi | 9176868900

Page 17

Diya Coaching Centre for Nurses One candle wipes out darkness…….

A. Stop the transfusion B. Notify practitioner C. Administer antihistamine D. Administer anti-inflamatory medicines 119. Which of the following types of health care is an example of primary level of care? A. Diagnosis B. Acute care C. Restoration D. Immunisation 120. In planning for the care of patient with Crohn’s disease the nurse and the patient discuss the interventions. Which of the following treatment modalities would most likely be considered a primary intervention for this disease? A. Surgery B Medication C High residue diet D. Blood replacement 121. A patient with acute Crohn’s disease has been prescribed an elemental diet. The most likely rationale for this is to A Reset the bowel B Improve nutrition C Improve medication absorption D. Prepare for surgery 122. A patient has a 6 year history of inflammatory bowel disease that is resistant to medical therapy. The patient can BEST decreased the likelihood of the disease progressing to A. Consuming only elemental foods. B. Stopping smoking C. Using effective birth control D. Avoid over-heating 123. A home health nurse is setting up a medication administration schedule for an elderly patient. The patient is taking oscul (calcium carbonate), Feosul (ferrous sulphate) and orazine (zinc sulphate). The patient meals at 8 am 12 noon and 6 pm. Which of the following medication sdministration times would the nurse most likely implement for this patient? A. Oscal, orazine, feosul at 8 am B. Oscal at 8 am, orazine at 12 noon and feosul at 6 pm C. Oscal and feosul at 12 noon and orazine at 6 pm D. Orazine at 8 am, oscal at 12.00 noon and feosul at 6 pm 124. A community health nurse is administering tuberculine skin tests purified protein derivative, which of the following time frames should tell the patient to return to the clinic for the test to he read? A. In 12-24 hours B. In 24-36 hours C. In 36-48 hours Compiled by : V.Devi | 9176868900

Page 18

Diya Coaching Centre for Nurses One candle wipes out darkness…….

D. In 48-72 hours 125. A patient who is scheduled for a tonsillectomy is in pre-operative units.The nurse notes an order for pre-anesthesia medication to be given on call to operation room. The nurse should give this medication A. Immediately upon being notified to prepare the patient for transport B. When the operation room staff arrives to transport the patient C. Only if clearly needed after assessment. D Upon the patients arrival in the operation room 126. A patient recently underwent coronary artery bypass graft (CABG) surgery. The nursing diagnosis includes sleep deprivation related to intensive care environment. The goal for this diagnosis would be that the patient A. Gets 4 hours of uninterrupted sleep during the night B. Takes naps during the day C. Is free of pain in the first hour post surgery D. Ambulate 3 hours post surgery 127. The nurse is assisting a patient to ambulate in the hall. The patient has a history of coronary artery disease(CAD) and had coronary artery bypass graft surgery (CABG) 3days ago. The reports chest pain rated 3 on a scale of 0 (no pain) to 210 (severe pain). The nurse should first A. Determine how long it has since the patient last dose of aspirin. B. Obtain a chair for the patient to sit down C. Assess the patient’s radial pulse D. Ask the patient to take several slow deep breaths 128. Which of the following discharge plans take priority for a patient who underwent coronary artery bypass A. Instruct on maintenance of low cholesterol B. Instruct on recognizing signs and symptioms of complication C. Instruct on the importance of maintaining physical excercise. D. Instruct on occasion of tobacco use 129. A urse is providing care to the patient having new skin graft on the left leg. The patient is upset and the nurse notes copious red drainage oozing around dressing. The nurse should immediately? A Lift the dressing to assess the area. B. Ask the patient is having any pain C Apply firm pressure for 10-15 minutes D. Assess the apical pulse 130. An elderly patient had surgery 2 days for intestinal obstruction vital signs at 10:00 am are temp 37.50 C HR-86, BP-132/72 pain level of 4 of 0 to 10. Abdominal dressing done is dry and intact. The nasal gastri tube to low intermittent suction. The pt is on strict input and out put every 2 hours. At 12.20 pm the patient complains abdominal pain, upon assessment the vital signs are T:37.5 C HR-98, RR-24, BP-146/88 pain level 8 out of 10. The patient abdomen is distended and rigid the dressing is dry and intact. The nurse should first:A. Reposition the pt on Rt side B. Irrigate the nasal tube to check for patiency C. Medicate the pt for pain as ordered Compiled by : V.Devi | 9176868900

Page 19

Diya Coaching Centre for Nurses One candle wipes out darkness…….

D. Increase the suction on his nasal gastric tube ti high intermittent suction 131. The traction and the urinary catheterhas been dis continuedfor a patient who was immobilized in traction for weeks. The patient is now having a problem with urin aryincontinence. Which of the following interventions would the nurse most likely implement? A. Behavioral training B. Bladder training C. Scheduled following D. Prompted voiding 132. To reduce the risk of treatment methicillin resistant staphylococcus aurevs (MRSA) from an infectious wound which of the following standard precautions should be implemented? A. Airbone B. Contact C. Droplet D. Reverse isolation 133. A patient experiencing intermittent claudicating in the legs while at rest which of the following nursing actions should the nurse takes? A. Vigorously massage the legs. B. Place ice on the ankles every 20 minuts C. Elevate the leg to the heart level D. Position the leg in the dependent position 134. The nurse is caring for a patient with chest tube connected to closed suction. The nurse should make sure that which of the following remains readily available at the patient bedside? A. Sterile towel B. petroleum gauze pads C. Normal saline solution D. Sterile gloves 135. While preparing, preperative paper work for a patient scheduled for neurosurgery, the nurse asks about the patient’s use of medications. The patient reports taking an aspirin tablet every day. But has not taken it today. The patient has had nothing by mouth since midnight of the day before, the nurse should. A. Inform the enesthesiologist innediately B. Tell the patient the surgery must be rescheduled C.Record the information on the form in red ink D. Obtain a blood sample and notify the attending physician 136. A 35 yrs old female has an inherited gene mutation for achondroplasia an autosomal dominate genetic disorder. Her husband does not have gene mutation. In planning genetic counselling for this pt the nurse would be most correct in including which of the following statements regarding the risk of their children inherited the genetic mutation? A.Each child has a 50% chance of inheriting gene mutation B. Female children have 50% chance of inheriting gene mutation C. Male children will not inherit the gene mutation D. All female children will inherit the gene mutation Compiled by : V.Devi | 9176868900

Page 20

Diya Coaching Centre for Nurses One candle wipes out darkness…….

137. A pt is one day P.OP repair of a large umbilical hernia. The pt complaints of abdominal pain and described feeling the sutures give way. Upon assessment of the abdomen the nurse observe an evisceration. The nurse immediate response should be to? A. Medicate the pt for pain B. Instruct the pt to cough hard C. Have patient perform valsalva manuver D. Cover the abdomen c sterile soaked dressing 138. A 3 yrs old child is seen at the pediatrician’s office. The parents reports the child has had vomiting and diarrhea per past 15 hours. The child’s is ie thragic with the following vital signs. Temperature 37.2 0c (99.00f). Heart rate 145, respiration rate 25, and blood pressure level 195/55mmhg. Which of the vital sign is abnormal A. Temperature 37.2 0c (99.0f) B. Heart rate 145 C. respiration rate 25 D. Blood pressure level 95/55mmhg 139. A home health nurse is teaching a family member about the care of a patient’s peripherally inserted central catheter (PICC) which of the following would be appropriate for the nurse to make? A. Place the use intravenous tubing in a leak proof container and then this seated container inside a secod leam proof container. B.You will need to put on a disposable face mask before you connect the intravenous tubing to the pott of the PICC. C. The port of the PIC catherter will need to be cleansed with betadine after the infusion is compleated. D.The empty medication container can be placed in the same container as your house hold reffuses. 140. While obtaining the pre operative history of a patient schedules for cosmetic surgery, the most valuable skill at the nurse disposal is A. knowledge of the procedure B. Time management skills C. Listening skill D. Empathy 141. A community health nurse screens a group of high risk adults for tuberculosis. Which gauge needle should the nurse use for an intradermal injection on the volar surface of the forearm. A. 16 gaugue needle B. 20 gaugue needle C.22 gaugue needle D.26 gaugue needle 142. A patient hospitalized with tuberculosis (TB) has a productive cough and hemoptysis. Which of the following types of isolated room would be the best choice for the patients? A. reverse isolation. B.Standard isolation C. Positive- pressure D. Nagative pressure Compiled by : V.Devi | 9176868900

Page 21

Diya Coaching Centre for Nurses One candle wipes out darkness…….

143. A patient diagnosed with tuberculosis is prescribed isoniazide (isoniazid) rifampin (rifadin) phyrazinamide (rifamate) ethambutol HLL (Myambuon) and acetaminophen. One month later the patient with hepatitis which drugs is the most likely cause? A. Ethambutol (myambuton) B. Acetaminophen C. Isoniazid (Isoniazid) D.pyrazinamide(Rifamete) 144. A patient with tuberculosis can transmit disease to another individual though; A. Air droplet B. physical contact C. Hand to mouth exchange D. Blood and body fluids 145. A patient recently underwent joint replacement surgery. Which of the following nursing diagnosis takes PRIORITY? A. Risk for peripheral neurovascular dysfunction B. Deficient knowledge on appropriate activity precaution C. Impaired physical mobility D. Sexual dysfunction related to pain 146. The parents are anxious after the doctor tells that child needs surgery. The assess parents ability to cope with this anxiety. Which of the following questions should the nurse ask A. Dis you know that feeling anxious about your child’s surgery is normal B. Can you wait until after surgery to begin to cope with being anxious? C.How do you think feeling of anxiety will affect your child? D. What has helped you when you felt anxious in the past? 147. 4 years old child brought to the community health clinic for schesuled immunization The child shiuld receive A. Varicella, rotavirus, pneumococcal and hepatitis B B. Measles, mumps, rubella and varicella C. Rotavirus and inactivated poliovirus D. varicella and haemophillus influenza 148. The nurse is conducted a community based educational program about diabetes mellitus. Which of the following statements by a participant would indicate correct understanding of the teaching? A. lantus insulin can be mixed with other insulin B. It is necessary to wipe off the top the insulin vial with alcohol to prevent infection C. Insulin will changed color after opening D. Needles can be placed in a hard plastic container with a tightly secure lid 149. A child is treated for superficial (first degree) thermal burns to the thigh The child is in great discomfort and does not eat. Which of the following diagnosis should receive PRIORITY? A. Altered nutrition B. Impaired skin integrity C. Risk for infection D. Acute pain Compiled by : V.Devi | 9176868900

Page 22

Diya Coaching Centre for Nurses One candle wipes out darkness…….

150. The nurse calls together an inter disciplinary team with members from medicine, social service and nutritional service to care for a patient with a terminal illness. Which of the following type of care the team most likely is providing A. Palliative B. Curative C.Respite D.Preventive 151. A nurse makes ahome visit to a patient recently diagnosed with chronic obstructive pulmonary disease (COPD) which of the following should the nurse teach the patients about managing COPD? A. Recognizing signs of impending respiratory infection B. Limiting fluids intake minimize bronchial secretion C. Correct technique to auscultate the lung fields D. Importance of starting antibiotic therapy 152. A patient with chronic obstructive pulmonary disease (COPD) experiencing frequent episodes of severe dysprea which of the following excercise would teach the patient how to be better control breathing? A. Lower side rib B. Segmental C. pursed lip D. Diaphragmatic 153. A patient with copd is experiencing frequent episodes of severe dysphnoea. Which of the following breathing execises would the nurse most likely implement? A. Lower side rib B. Segmental C. pursed lip D. Diaphragmatic 155. In evaluating the appropriateness of various excercises enjoyed by a patient with osteoporosis the nurse would recommend A. walking B. Bowling CSit-ups D. Golf 156.A patients to the clinic with a “ pins and needles” sensation of the left foot and complaints that objects appear ‘ Shimmering”. The patients is diagnosed with optic neuritis and reffered for further testing. The patient is MOST likely to be tested for A. Glaucoma B. Multiple sclerosis C. Lesion of brain stem D. sychosis 157. A 3 years old has returned to the clinic 4 days after being diagnosed with gastro enteritis and dehydration A parent reports that the vomiting has stopped and the child is tolerating liquids, rice, apple sauce and bananas. The diarrhea persists, but seems to be decreasing in volume when evaluating for signs of dehydration, the nurse will assess the patient’s skin turgor by Compiled by : V.Devi | 9176868900

Page 23

Diya Coaching Centre for Nurses One candle wipes out darkness…….

A . grasping the skin over the abdomen with two fingers raising the skin with the two fingers. B. Grasping the skin over the forehead with two fingers and raising the skin with two fingers C. Holding the patients mouth open and assessing the tongue for deep creases or furrows. D. Drawing two tubes of blood and running blood urea nitrogen and creatine. 158. A patient is admitted for pain management due to lung cancer with metastasis to bone with a nursing diagnosis of alteration in comfort the nurse would anticipate the best short term goal would be to; A. Not complaint of pain B. Appear comfortable and sleep well C. Verbalise that pain is releaved D. verbalised that pain is tolerated 159. A nurse is assessing a paetient who just arrived in the emergency department after amotor vehicle collision. The patient has astrong smell of alcohol on the breath, is restless, and has abluish discoloration on the abdomen by the umbilicus the patients vital signs T:37:I20 Beats/min RR:24/min and BP:100/62 mm of hg While other members in the team are evaluating the patient the nurse should obtain A. A pair of elastic support stocking B. A chest tube insertion tray C. Supplies for peritoneal alvage D. A vial of Hydralazinc 160. While caring for a terminally ill preschool aged child whose death is not eminent, the child asks the nurse, am I going to die? The best Nurse response is A. I’m not sure what is wrong with you, but i hope not B. Don’t worry when you die you will be angels C. We all die someday, but you are not going to die today or tomorrow D. I can’t talk to you about that, you will have to ask your doctor 161. A patient with chronic obstructive pulmonary disease complains of a frequent cough, bilateral wheezing is auscultated in the lung fields The nurse administers albeterol Neubulizer treatment as ordered and educates the pt on way to decrease exacerbation. Which of the following action indicates that the patient understand instruction? A The patient reduces numbers of cigarettes smoke day B. The patient requested a pneumococcal vaccination C The Patient increases sodium and potassium intake D. The patient exercise whenever experiencing shortness of breath 162. The nurse administers albeturol nebulizer to a child with Asthma exacerbation. The nurse measures pulse oximetry and auscultates the lungs to determine whether the goal of clear respiratory status has been met. The step of nursing process is called ? A. Assessment B. Diagnosis C. Implement D. Evaluation

Compiled by : V.Devi | 9176868900

Page 24

Diya Coaching Centre for Nurses One candle wipes out darkness…….

163. The home care nurse observes that the asthmatic patient has a cough wheezing. The nurse administers an albuterol (provenul) nebulizer treatment as ordered. Which type of implementation is this? A. Discharge planning B. Instruct C. Monitoring and surveillance D. Therapeutic interventions 164. A child with asthma has an order for albuterol prior to administration of the medication the nurse must: A.Pre-oxygenate the patient B. Obtain venous access C. Assess the patient’s heart rate D. Feed the patient a snack 165. When administering albuterol to a child with asthma the nurse should bserve for signs of what major side effct t this mediction? A. tachycardia B. renal failure C. pneon D. blurred vision 166. child with sthm is experiencing hick respiratory secretions resulting in increased wrk f brething the best nursing intervention is to; A. Encourge fluids B. Eliminate dairy products C. Decrease reltive humidity f the rm D. make the child ly n the left side E. Eliminate fluids 168. What would be he long term golf r child with asthma? A. Quickly reverse ir flw obstruction B. correct hypoxemia C. deliver humidified oxygen vi nasal cannula D. Develop a home and school mngement plan 169. The nurse administers an albuterol nebulizer on a child with asthma exacerbation. When of the following indicates effectiveness of treatment? A. dventitious breth sound with cough B. O saturation C. Nasal flaring D. respiration rte 28 170. A child is dignosed with asthma exacerbation which of the following nursing diagnoses should be the first priority? A. ineffective airway clearance related to bronchospasm and mucosal edema B. Fatigue related to hypoxia C. Anxiety related to illness and loss of control D. Deficient knowledge related to potential side effect of the medication Compiled by : V.Devi | 9176868900

Page 25

Diya Coaching Centre for Nurses One candle wipes out darkness…….

171. An asthmatic patient presents with wheezing and coughing. Oxygen saturation is 88% on room air. Which of the following nursing diagnosis would take priority? A. Imbalanced nutrition related to decreased food intake B. Activity intolerance related to inefficient breathing C. Anxiety related dyspnea and concern of illness. D. Ineffective gas exchange related to bronchospasam 172. The nurse is visiting the asthmatic patient at home to reinforce the importance of eliminating environmental allergens and to assess the patient’s response to the environment al chnges. The type of implementation is cancelled A. Supervision and coordination B. Discharge plnning C. Monotoring and surveillance D. Therapeutic interventions 173. The parent of a child with chronic asthma is hesistant to discipline becuse the child doesn’t feel well. The nurse should encourage the pt to A. set consistent behaviour limits B. Be more lenent during time of illness C. Cherish the limited time the child has to live D. void upsetting the child with limit setting 174. In developing the care plan for hospitalized 3 years old child with asthma the nurse plan to talk calmly in an appropriate language and explains all procedures. Which of the following statement by the best demonstrates implementation of the approach? A.You can use stethoscope to listen to your heart rate and your dolls and then I’ll listen B. You must not wiggle while I listen to your heart you can hold your doll C. The stethoscope will feel cold on your chest you can tell your doll how cold it is D. Let’s go for your doll and place your hands on your tummy while I use stheth. 175. A nursing process which involves the performance of a nursing plan care is called A. Assessment B. Nursing diagnosis C. implementtion D.Evaluation 176. A ptient who is receiving chemptherpy hs a platelet count of 49,000mm cube(norml value 1,50000 to 4000,00) which of the following nursing action is necessary A. Minimize invasive procedure B. Crush oral mediation C. limit intake of vit.k rich foods D. monitor the temperature every 4 hourly 177. An elderly patient with long history of diabetic mellitus comes in for a routine checkups. Which of the following nursing diagnosis would be the nurse anticipates? A. Risk for impaired skin integrity related to decrease sensation and circulation B. Excess fluid volume related to disease process C. Risk for injury to decrease gastric mobility and stress response D. Deficient fluid volume related to diarrhea and loss of fluids and electrolytes? Compiled by : V.Devi | 9176868900

Page 26

Diya Coaching Centre for Nurses One candle wipes out darkness…….

178. A 3 yrs child is brought to the office by the prents who have been toilet trinig the child for the past 5 months with little success. The parent has been using rewards for keeping the child clean and dry. Today the prent relized that the child abdomen was very firm, the appetite was poor, and there had not been bowel movement for 6 days with nursing diagnosed of alternation in bowel elimination, what is the best goal? A.A child will recognize the urge to defecte dily B. The prent will use prise when the child defets in the toilet C. Predicable, regular bowel hbits will be restored and maintained D. Toilet training will be delayed. Until the child is congnively ready 179. The nurse is teaching a patient bout spironolactone (aldactone)... which of the following instructions should the nurse review with the patient? A. Increasing intake of foods that are high in potassium B. Taking the medication right before going to sleep C. Avoiding seasoning that are labeled as salt substances D. Scheduling the medication so that multi vitamin is taken an hour later 180. Which of the following best indicate effective treatment of meningitis with an intravenous antimicrobial agent? A.Severe hedche B. Negtive kerning sign C.Nuccal rigidity D. Photophobia 181. A 32 year old female comes in for evaluation 14 days after an uncomplicated cesrean section. The patient’s very anxious and complining of shrp stbling pain in her chest. The ptient has dyspnea, tachypnea and hypoxemia. Which of the following post operative complication is likely? A. Pulmonary embolism B.telectsis C.Pneumonia D. Aspiration 182. home care nurse reviews the laboratory results for a post partum patient who had acaesarian section. Which of the following indicates possible wound infection? A. Incresed WBC B. Decreased heamtocrit C. Increased haemoglobin D. Aspiration 183.Three days ago patient underwent an invasive surgery with an open wound. The patient is febrile with drop in blood pressure. Laboratory test result shows elevated while blood cell count. This could be possible A. Sepsis B. Atelectsis C. Internal haemorrhaging D. Excess fluid volume

Compiled by : V.Devi | 9176868900

Page 27

Diya Coaching Centre for Nurses One candle wipes out darkness…….

184. A conscious victim of motor vehicle accident arrives at the emergency department. The patient gasping for air, is extremely anxious, and has a deviated trachea, what diagnosis should the nurse anticipate? A.Pleaural effusion B. Tension pneumothorx C. Pneumothorax D. Hemothorax 185. ptient brought to the emergency room with severe head injury. A craniotomy is performed to evaluate ablood clot. Which of the following is desired expected out comes 24 hour post operatively? A. Gg reflex present B. cerebrl perfusion pressure, 68 mmhg C. Intracrnial pressure, 2 Immhg D. Decreased lacrimation 186. A nurse is assigned to patient who is scheduled for n above knee mputation of the left leg. During the pre-operative preparation, the nurse should ask the patient to? A. Write yes on the leg B. Write on the right leg C. Draw on arrow on the left knee pointing upward D. Draw an arrow on the left knew pointing downward 187. A patient who is 18 hours post operative after an above the knee amputation complain of feeling like something is crawling under the dressing as well as increase pressure of the dressing. The nurse suspects hemorrhge. The patients vital sign remins within the normal range . What should the nurse do first? A. Call the physician B. Plce ice round the dressing C. Encourage patient to discus fears D. Lower the temperature of the room 188. patient finds their 2 weeks old infant unresponsive. Infant is limp, Cyanotic and pale there is no respiration while the skin is cold to touch. The parent begin resuscitation and is the infant transferred to the hospital where the infant expires. The most likely diagnosis is A. Sudden infant death syndrome B. Apparent life-threatening event C. Apnea of infancy D. Apnea of unknown origin 189. A neonatal nurse performs Apgr assessment at 1 minute of breath to evaluate the physical condition of the newborn and immediate need for resuscitation at 1 minute. Apgar scores 7, at 5 minutes Apgar score is 10, the progression of scores suggests. A. healthy newborn B. The need for supplement oxygen C. genetic defect D. The infant is becoming stable Compiled by : V.Devi | 9176868900

Page 28

Diya Coaching Centre for Nurses One candle wipes out darkness…….

190. The nurse is caring for full term newborn who was delivered vaginally 5 minutes ago. The infant’s APGR score was 8 at one minute at 1 at 5 minute. Which of the following has the highest priority? A. Maintaining the infant in the supine position B. Assessing the infant’s red reflec C. Preventing het loss from the infant D. Administering humidified oxygen to infant 191. which of the following can be use determine if a prescribed pain management therapy is effective for anon verbal patient? A. rest well B. taking a sleep C. talking too much D. feeling good 192. While caring for aneonate with a meningocole the nurse shuld avoid positioning the child on the A. Abdomen B. Left side C. Right Side D. Back 193. A patient with exacerbation of CHF has a nursing diagnosis of excessive fluid volume. The nurse monitors fluid intake and output and administered furesemide s ordered .Which of the following indicates the efficiency of the intervention. A. the patient has pitting edema B. the patient has shortness of breath C. the patient has a decrease in weight D. the patient has jugular vein detention 194. A 62 years old has treated for patient congestive heart failure and a nursing. Diagnosis of fluid volume excess. After diuretic therapy and dietary interventions, the patient has met all short term goals. The nurse should A. Revise the care plan with diagnosis of risk for alteration in fluid balance B. add a new diagnosed of risk of fluid volume deficit C. discontinue the care plan as the diagnosis is resolved D. Continue the care plan as written 195. A patient with congestive filure nd sever peripherl edem has nursir diagnosis of fluid volume excess. What re the two most important interventions for nurse to initiate? A. Diuretic therapy and intake and output B. Nutritional education and low sodium diet C. Dily weights of intake output D. Low sodium diet and elevate legs when in bed 196. patient has exacerbation of CHF which one of the nursing diagnosis beir excess fluid (lasix). Which of the following indicates the efficacy of the nursir intervention. A. The patient has leg edema B. The patient has shortness of breath Compiled by : V.Devi | 9176868900

Page 29

Diya Coaching Centre for Nurses One candle wipes out darkness…….

C. The patient as decreased in weight D. The patient has jugular vein destention 197. When caring for a ptient with ostomy, the nurse knows that extra ask protection for peristomal skin is most important for those with a (n); . scending colostomy B. Trnsverse colostomy C. Ileostomy D. Sigmoid colostomy 198. While evaluating the nutritional intake of a bedridden patient with multiple pressure sores, the nurse should make sure the patient increase the tke of A. Protein rich foods B. Water C. Food rich in vitamin A D. fiber rich foods 199. In what position should dyspnoeic patient be placed? A.Prone B.Recumbent C.Semi-fowler’s D. Trendelenburg 200. A patient presents to the clinic for routine visit and has the following vital sign temperature 37 0 C (98.6 0f) HR82 respiration rate 18, and BP Bo/94mmhg which vil sign is abnormal? A. Temperature B. Pulse C. Respiration D. Blood pressure 201. femle patient dmitted for bdominl pin complains of generalized paiNuse, Vomitting and constipation, nursing assessment fins; temp 38.6 0 c (101.5 0 f) heat rte 92, RR 18, BP 130/68 mmhg. The patient has rebound tenderness and bdomin rigidity in the pst hour. Her pain has localized on the RT side., The nurse suspects, A. Intestinal obstruction B. Influenz C. Appendicitis D. Pyloric stenosis 202. community health nurse is implementing an dult immunization program the neighbourhood, which of the following would most likely be universl recommended sdult vaccination and does frequency general population A. Tetanus-diphtheria toxoid every 20 years B. Pneumococcal vaccination every 2 years C. Influenza vaccination every year D. One time typhoid vaccine followed by boosters every 5 years 203. A 6 old patient has been diagnosed with cute rheumatic fever. Antibiotic for this illness is? A. Bezthgine pencillin (megacillin) B. Amozicillin Compiled by : V.Devi | 9176868900

Page 30

Diya Coaching Centre for Nurses One candle wipes out darkness…….

C. Erythromycin D. Vancomycin 204. A child is admitted to the hospital with congenital heart disease. Which of the following nursing diagnosis should receive priority? A. Decreased cardiac output related to decreased myocardial function B. activity intolerance related to carbexi C. Impaired gas exchange related to altered pulmonary blood flow. D. Imbalanced nutrition less than body requirements related to excessive energy demands. 205. Prior to initiating therapy with infractionted heparin for a patient hospitalized with deep vein thrombosis the nurse should plan to A. Weight the patient B B. dminister spirin C. Limit fluid intke D D. Undress the patient 206. Prior to initiating therapy with unfractionated heparin for a patient hospitalized with a deep vein thrombosis this treatment requires. A. Bed rest B. spirin therpy C. Fluid restrictions D. A high protein diet 207. A patient with chronic liver disease secondary to patients C has been admitted with a nursing diagnosis of alternation in nutrition. Less than body requirement the BEST long term goal is the patient will. A. gin at least 10% of body weight B. Attiain nd mintin ideal weight C. Verbalize understanding nutritional needs. D. Include high quality protein in diet. 208. The nurse is Assessing a patient with a history of seizure disorder. While checking the patient vital sign the patient develop rhythmic, jerking movements of the arms and legs. The nurse should immediately place the patient in which of the following positions? A. Prone B. Supine C. Semi-Fowler’s D. Lateral 209. nurse is ssessing to care for a child aseizure disorder the nurse observe the child becomes stiff and lose consciousness. Following by jerking movements for 1minute after which the child becomes very sleepy. Which of the following types of seizures occurred? A. Absence (petit mal) B. Generalized (tonic conic) C. partial psychomotor (temporal lobe) D. Status epilepticus 210. Patient is scheduled for an abdominal neurysm repair. This is what type is what type of surgical intervention? Compiled by : V.Devi | 9176868900

Page 31

Diya Coaching Centre for Nurses One candle wipes out darkness…….

A. Diagnostic B. Transplant C. Curative D. Palliative 211.A community health nurse is teaching heath. Class about infectious disease. Process the nurse class that Rabies would be considered which of the following types of infection? A. Viral B. Protozoan C. Fungal D. Bacterial 212. A patient receiving chemotherapy developed some raised red.Edematous wheel on the skin. Which of the following care plan alternatives would most ikely need to occur before the next treatment? A. Reinforced relaxation techniques B. Cntinue chemotherapy without change C. Continue with radiation therapyonly D. Premiditate the patient with an antihistamine 213. A 6year old patient has presented to the cinic with fever, malaise and anorexia, the patient was treated 2 weeks ago for streptococa infection of the throat. The nurse should expect the physician to order what test? A. Electro cardiogram B. Jones test C. Spinal tap D. Heart biopsy 214. A community is experiencing an outbreak of staphylococal infections. The nurse instructs residents that the most common mode of transmission is by: A. Respiratory droplets B. Contaminated foods C. Hands D. Soil 215. A hospitalized patient has fallen from bed, the nurse notes shortening of the left leg. Pain upon movements of the left leg and rapid swallow respirations what action should the nurse take First? A. Call for help B. Immobilize the left leg C. Obtain blood pressure D. Evaluate lung sounds. 216. A community health nurse visit patient who was suffered a stroke. The patient spore explain to the nurse that the patient chokes on foods at times which of the following referral ordered would the nurse anticipate needing for this patient? A. Speech therapist B. Dietician C. Physician therapist D. Neurologist Compiled by : V.Devi | 9176868900

Page 32

Diya Coaching Centre for Nurses One candle wipes out darkness…….

217. A 59 year old patient arrives in the emergency department diaphoretic and complains of chest pain and shortness of breath the patient is sibling states that this has happened before and it is just anxiety. Upon evaluation the physician diagnosis result of this drug therapy? A. Balanced between oxygen supply and demand B. Increase in blood flow to the heart C. Reduction in oxygen demand and consumption D. Vessel relaxation 218. A patient with end-stage cardiomyopothy and angina pectoris to the office complaining of frequent chest pain and severe dyspnea with a nursing diagnosed of alteration in comfort. What is the BEST short term goal for the patient? A. Perform all activities of daily living without complains of chest pain or shortness of breath B. Verbalize and employs strategies to decrease pain and increase coronary blood flow C. Take pain medications around the clock and use supplement oxygen at all times. D. Understand the disease process and accept the imitation that it place on his life style 219. A patient has an order for a pneumatic compression device. Which of the following is an appropriate goal? A. Reduce the risk deep vein thrombosis B. Reduce lower extremity edema C. Reduce lower extremity pain D. Reduce the risk of phlebitis 220. A patient with severe diverticulitis had surgery for placement of colostomy. The patient upset crying and will not look at the colostomy. Which of the following would be the HIGHEST priority Nursing diagnosis at this time? A. Knowledge deficit, colostomy care B. Distorted body image C. Self case deficit, toileting D. Alteration in comfort 221. A patient came to the emergency department with complaints, dizziness and confusion; clinical symptoms include tachypnea and dyspnea with the use of accessory muscles to facilitate breathing. Which of the following orders should the nurse most likely implement to reduce the patients confusion and disorientation? A. Oxygen therapy B. Chest physic therapy C. Bronchodilator D. Hydration fluids 222.A 6-month old boy is admitted with a diagnosis of failure to thrive. According to the growth chart at month of age the infnt’s weight is in which percentile? A. 25 th B. 5 th C. 10 th D. Below the 5 th

Compiled by : V.Devi | 9176868900

Page 33

Diya Coaching Centre for Nurses One candle wipes out darkness…….

223. A patient is 2 days postoperative hernia repair and has an order for a dressing change, patient has been diagnosed with immune deficiency disease syndrome (AIDS). While performing the dressing change the nurse shoud take which of the following actions? A. Put the patient in a private room. B. Wear gloves during the dressing change C. Wear gloves, gown, mask during the dressing change D. Put the patient in reverse isolation 224. When administering an enema to adult patient. How far should the nurse insert the tubing into the rectum? A. 2.2 to 4.4 cm (1 to 2 inches) B. 4.4 to 6.6 cm (1 to 3 inches) C. 6.6 to 8.8 cm (3 to 47 inches) D. 8.8 to 10.1 cm (4 to 5 inches) 225. A nurse is implementing Nursing interventions to monitor a patient following Kidney surgery. Which of the following complications would be the most likely post operative risk after renal surgery? A. Deep vein thrombosis B. Hemorrhage C. Nausea D. Hemiparesis 226. As a part of Neurological assessment which of the following associated with the highest score on the Glasgow coma scale? A. Eye opening to pain no verbalisation B. Confused obey commands C. Eye opening to speech confused D. Localized pain abnormal extension 227. While caring for a patient prior to surgery to amputate the leg. What is the most effective measure to prevent phantom limb sensation after the amputations? A. Cntrol pain prior to the surgery B. Make sure the patient understands the preocedure C. Elevate the imb on two pillows D. Help the patient grieve for the limb 228. If a patient develops a complication during a blood transfusion the nurse first action should do to A. Stop the transfusion B. Notify the practitioner C. Birth control pills D. Help the patient grieve for the limb 229. A patient n elevated prothrombin (PT) time which medication should the nurse consider as a possible cause of the elevated PT time? A. Rifampin B. Vitamin K C. Birth control pills Compiled by : V.Devi | 9176868900

Page 34

Diya Coaching Centre for Nurses One candle wipes out darkness…….

D. Phenytoil 230. A home care nurse visit a patient with a new ‘below the knee amputation’. The cite of the incision is red, warm and tender with purulent yellow drainage. The pt has a new prescription for cephalexin (keflex) nd oxycodone(oxycontin). What would the nurse instruct to do FIRST ? A. Take oxycodone as soon s possible B. Take cephalexin as soon as possible C. Wash the incision site and apply bacitracin cream D. Wash the incision site and apply hydrocortisone 231. A patient has the following order cephalexin (keflex) 500mg. By mouth 4 times day, the pharmacy has the following dose 250 mg. Per 5ml. The nurse should administer A. 5 ml B. 10 ml C. 15 ml D. 20ml 232. A marathone runner experiences a sudden onset of sharp pain in the calf immediately after a workout. Which of the following would the nurse suspect the patient is experiencing? A. Bursitis B. Tendonitis C. Plantar facitis D. Joint dislocation 233. A mastectomy ptient has develop[ed lymphedema of the eft arm. The Nurse shoud teach the patient that hte bst position for the arm is A. Immobilised across the chest B. Dependent C. Elevated D. In traction 234. A patient in the emergency room 20 cm laceration with the right forearm. The nurse prepare for which type pf anesthesia to be administered before the laceration is repeated by the physician? A. Intravenous B. Regional C. Local D. General 235. A nurse in a community health clinic is in change of immunisation. When patient visits the cinic, the nurse knows that immunisation should be reviewed. A. At the age they are scheduled to be administered B. One month prior to recommended immunization schedule C. At every clinic visit D. At monthly intervals 236. A child was admitted to the hospital three hour ago with a closed head injury. The child responds appropriately but sluggishly to stimuli and drift in and out of sleep. Which of the following best describes this patients level of consciousness ? A. Lethargic B. Obtunded Compiled by : V.Devi | 9176868900

Page 35

Diya Coaching Centre for Nurses One candle wipes out darkness…….

C. Semi comatose(drowsiness) D. Comatose 237. A healthy patient is in the doctor’s office for a pre-operative visit before a total replacement. The nurse interviewing the patient charts, the following medications. Aspirin 81mg once a day, Vitamin E- 260 international un its once a day and unknown amount of a herbal supplement once a day. Based on the p-atient medications list, which of the following labs would be important prepaeratively? A. Prostrate specific ntigen(PSA) B. Blood glucose C. Creatine phosphokinase iso enzymes(CPK enzymes) D. Prothrombin time. 238. A patient with long standing DM (type 1) is scheduled for surgical amputation of 4 gangrenous toes on the right foot. Which surgical intervention would this be classified as? A. Paliative B. Curative C. Reconstructive D. Diagnostic 239. The nurse is caring for a patient who just had chest tube inserted due to a spontaneous pneumothorax. An appropriate goal is that the patient will A. Be free of pain within 4 hours B. Report decreased pain C. Rest quietly D. Sleep with few movements 240. A patient with the deep vein thrombosis (DUT) is being treated with a ow molecular weight heparin (LMWH) the patient reports increased pain in the affected extremely. The nurse observes the affected extremey has increased in size bu 0.2 cm (0.8 inches) during the past 24 hours. Which of the following actions should the nurse take? A. Administer the next dose of LMWH before the scheduled time. B. Apply dry heat to the site. C. Elevate the extremity D. Reinforce the importance of ankle circling Exercises. 241. A physician orders a lactated ringer solution to infuse at 125 cc/hr. This is an example of which type of solution? A. Hypotonic B. Hypertonic C. Isotonic D. Hyper alimentation 242. Physician order an intravenous fluid of D5NS at 100.cc /1 hr. This is example of which of the solution ? A. Hypotonic B. isotonic C. hypertonic D. Hyper alimentations Compiled by : V.Devi | 9176868900

Page 36

Diya Coaching Centre for Nurses One candle wipes out darkness…….

243. A patient is in the pre operative area to Lumbar surgery. The patient reports anxiety bout being intubated and expresses concern about waking up during the surgery. The nurse must discuss the patient corcern with the A. Anesthesia Provider B. Surgeon C. Scrub nurse D. Charge nurse 244.The nurse is caring for a patient dignosed with Human Immune Deficiency virus. Which of the following nursing diagnose take priority? A. Diarrhea related to medication side effects. B. Risk for infections related to inadequate immune system. C. Imbalanced nutrition related to decreased apetite D. Impaired tissue integrity related to cashexia and malnourishment 245. A nurse access a 3 month old infant. The patient expresses and feeling over whelmed. The nurse offer information on available parenting support. This level of child abuse prevention is classified as which of the following? A. Intervention B. Primary C. Secondary D. Tertiary 246. The nurse is caring for a patient with a coronary thrombosis who is receiving prescribed streptokinase (Streptase). The patient reports the onset of rash as well as feeling hot while experienced chills. The nurse should immediately implement the plan of care for ? A. Medication side effects B. An allergic reaction C. A pulmonary embolus D. Perphera artery occlusion 247. The nurse is teaching a patient who was just diagnosed with narcolepsy. The nurse should teach the patient that which of the following typically increases the level of fatigue. A. Taking brief Naps B. Participating in an exercise programme C. Eating large meals D. working in a co environment 248. The physician has prescribed quinidine poyglacturonate (Apo-Quinidine 8.25mg, every 4 hours for a patient who weighs 50 kg. The drug is available as a 275 mg tablet. The nurse should administer how many tablets for each doses? A. 2.5 B. 2 C. 1.5 D. 1 249. The nurse is teaching the parent of a child with celiac disease. Which of the following diets should be revival with the patient? A. Guten-free Compiled by : V.Devi | 9176868900

Page 37

Diya Coaching Centre for Nurses One candle wipes out darkness…….

B. Dairy free C. Vegetarian D. Sodium restricted 250. A patient has peripheral vascular disease The nursing diagnosis is ineffective tissue perfusion peripheral, which of the following is n appropriate goal ? A. Patient wil identify three factions to improve peripheral circulation B. The patient will have palpable peripheral pulses in one week C. The patient’s feet will be warm to touch D. The patient will ambulate the length of the hallway 251. On the second day of hospitalisation for verticuloperifoneal shunt revision, a child with spinal bifida developed lier itching and wheezing. The nurse should determine if the patient has been exposed to A. Peanuts B. Strawberry C. Eggs D. Latex 252. A patient with malignant cancer has decided to stop chemotherapy and receive hospice care. What is the priority nursing diagnosis? A. Alternation in comfort B. Hopelessness C. Powerlessness D. Non-compliance 253. A nurse accessing a 16 month old child observes bruises scattered over the body that are at different stage of healing. The chid also has poor and diaper rash. The goal f treatment for this chid is to: A. Ensure the physical and emotional safety of the chid B. Remove the chid from the parent C. Admonish the parents of the child D. Ensure that the child stays with the bioogy 254. While visiting a patient who had a left hip replacement surgery one week ago, the patient complains to the home care nurse of episodic numbers and finding of the lower left extremities. Assessment of the patient shows that the lower left extremities are slightly cool to touch when compared to the lower right extremities. There is no swelling or redness on assessment. What would be the NEXT nursing intervention? A. Reassure the patient that this is normal after surgery B. Refer the patient to the surgeon immediately C. Encourage the patient to decrease activities involving the left hip and extremities. D. Refer the patient to a physical therapist immediately 255. A nurse is evaluating a patient 5 days after a right tota hip replacement. Which of the following goals is appropriate for the patient? A. Maintain hip abduction without dislocation B. Rest with legs elevate while sitting C. Tie shoes and put on undergarments without assistive devices Compiled by : V.Devi | 9176868900

Page 38

Diya Coaching Centre for Nurses One candle wipes out darkness…….

D. Perform scissors like leg exercise daily 256. Prior to providing care for a hospitalised infant, the nurse must: A. Check respiration B. Maintain a patient airway C. Check vital signs D. Hand washing 257.When caring for a patient with new sigmoid colostomy. The nurse knows that the stoma nods that increase in size? A. One month B. Two month C. Six month D. One year 258. A 7 week old infant boy is admitted with projectile vomiting decreased urine output decreased bowel movements and weight loss. He has poor turgor and appears hungry. The nurse observes left to right peristaltic waves after the vomits. The nurse would expect to find which of the following during the physical assessment? A. Hepatosplenomegaly B. A palpable pyloric mass C. Lymphadenopathy D. Bulging fontanelles 259. A nurse will need to change the dressing on a patient’s central venus catleter during the shift. The nurse would plan to A. Limit the patient’s activity for an hour dressing change B. Position the patient on the left side before removing the old dressing C. Put on sterile gloves after explaining the procedure to the patient D. Cleanse the insertion site using a circular motion 260. During the operative period, a nurse is assigned to care for a morbidly obese patient with an abdominal incision. The nurse knows that this patient’s weight increases the risk of A. Left sided heart failure B. Pressure sores of the coccyx C. Conspiration and ileus D. Wound dehisecence 261. Which of the following takes place during the implementation phase of the nursing process? A. Development of a goals and a nursing care plan B. Identification of actual or potential health problem C. Actualization of the care plan through nursing interventions. D Deformation of the patient’s responses to the nursing interventions 262. For a patient with a colostomy which of the following interventions is appropriate for preventing the risk of the impared skin related to exposure expressions? A. Empty pouch when it is completely full B. Remove the skin barrier inspect the skin monthly C. Recaps skin barrier opening to size of stoma with each change D. Cut an opening in the skin barrier then the circumference of the stoma Compiled by : V.Devi | 9176868900

Page 39

Diya Coaching Centre for Nurses One candle wipes out darkness…….

263. An infant who weigh 9kg (19.8165 require 900m of fluid per day for maintenance fluids. The infant typically consumes 120 ml during each feeding. The infant must have how many feedings per day to meet the fluid maintenance needs? A. 4 B. 8 C. 10 D. 12 264. A patient has a pacemaker implanted. Which of the following interventions is appropriate for the nursing diagnosis for risk for injury? A. Have pt avoid exposure to MRI B. Observe incision for redness, warmth, purulent discharge and soreness C. Offer backrubs to promote relaxation D. Instruct pt in dorsiflexion exercises of ankles 265. A. Pt undergoing treatment for cancer with bone metastasis is experiencing severe pain. Which of the following treatment the nurse most likely expect to improve patience pain control? A. Adjuvant radiation therapy B. Palliative radiation therapy C. Curative radiation therapy D. Radio surgery 266. A home care nurse visits a pt with diabetes. The pt ate 3 well balanced meals sweet desert and exercises 30 minutes a day twice a week. Also the pt takes hypoglycaemia medication and the blood glucose level ranges from 150-200mg/dl. The nurse set a goal of eliminating sweet dessert and increasing the frequency of exercises to three times a week. This week the patient exercised for three times a week for 30 minutes and ate dessert only after dinner. The glucose range from 100-150 mg/dl. The nurse evaluate that : A. The goal will not be met B. Progression is being made towards the goal. C. The goal is met D. The goal is inappropriate 267. A nurse is assigned to care for a patient with an ileostomy. The nurse would expect the ostomy discharge to be A. Fluid mushy B. Mushy C. Liquid D. Solid 268. A nurse educates a patient diagnosed with diabetes, on the importance of exercise and a well balanced, low carbohydrate diet. The patient takes metforin (glucophage) 500 mg once a day. which of the following indicates the patient’s plan of care needs to be re evaluated? A. Blood glucose level is 90mg/dl B. Hbg Aic (glychemogobin) level is 9.0% C. Total cholesterol level is0mg/dl D. Low density lipoprotein is 130mg/dl Compiled by : V.Devi | 9176868900

Page 40

Diya Coaching Centre for Nurses One candle wipes out darkness…….

269. A nurse schedules a patient for surgical procedure to take place in 1 week. When would the nurse Most likely implement surgical education? A. After admission to the hospital B. Start during the visit C. Immediately prior to anesthesia D. After the operation 270. The nurse is inserting a nasogastric (NG) tube into a patient as prescribed. The nurse has advanced the tube into patient’s posterior pharynx. The nurse should ask the patient to A. Hold the breath B. Stair upwards will eyes towards the ceiling C. Perform the valsalive mameuver D. Lower the chin towards like chest 271. A home care nurse staff a diabetic patient who was started on insuin injection upon examination, the nurse observe small limps and dents on the right upper where the patient has injectd insulin, what is the best nursing intervention ? A. Refer patient to dermatogist B. Refer the patient to endocrinologist C. With hold insulin D. No need of management 272. After cardiac surgery a pt have been prescribed low sodium, low cholesterol. Which of the following menu is the BEST ? A. Salami, dry bread B. Baked chicken thigh sliced tomatoes, ice berg lettuce C. Pasta with canned tomato sauce, wheat bread D. Bacon, lettuce and tomato sandwitch with mayonnaise dressing 273. A home helth nurse visit a patient with COPD using home oxygen 21/mt. The reports period of shortness of breath and enquires about increasing the oxygen to 41/mt. The nurse explains that the increasing supplemental oxygen wil A. Increase activity tolerance B. Suppress the hypoxic drive C. Activate the shortness of breath D. Prevent lung infection 274. The nurse should avoid the use of the dorsoglateal site for an intramuscular injection in children because of the risk for injury to which of the following nerves? A. Vagus B. sciatic C. Ilionguinal D. Lumbar plexus 275. Twelve hour after removal of a benign liver tumer, the nurse observed that the patient has decreasing blood pressure, decreasing pulse pressure increasing het and increasing respiratory rate. The patients skin is cool and pale after lowering head of the bed, what should the nurse do next? A. Cal the physician B. Administer pain medication Compiled by : V.Devi | 9176868900

Page 41

Diya Coaching Centre for Nurses One candle wipes out darkness…….

B. Position the patient on the left side D. Apply cool, wet clothes under the arm 276. The nurse is assigned to care for an elderly ptient with ow exudates pressure ales which if the following types of dressing woud the nurse most likely to use? A. Hydrogel B. Hydrocolloid C. Polyurethane D. Polyurethane foam 277. A patient with an unsteady gait and history of falls has a care plan inte that including keeping the walker in research and pathway free of obstacle evaluation after 1 week. The patient has had no falls but the gait remains un control. The nurse should A. Continue the plan of care as written B. Allow the patient to replace the walker with care C. Allow the patient to ambulate short distance without the walker D. Have the patient practice stopping over small objects 278. The nurse is caring for a patient who had a totl colectomy 24 hours ago due to a malignant neoplasm in the rectum. The patient continues to receive intravenous fluid and has started a clear liquid diet. The nurse understands that the patient is at INCREASED risk for which of the following postoperative complication? A. Disseminated intravascular coagalopathy B. Atelectasis C. Syndrome of inappropriate anti diuretic hormone(SIADH) D. Hypokalemia 279. When doing community based teaching for latex allergies, the nurse should plan to teach the patient that; A. Food handled by people wearing latex gloves stimulate an allergies responses B. Foods containing nuts may trigger n allergic cross response in people with latex allergies C. The patient should wear a face mask while in the hospital due to large amount of airborne latex D. Houses used on gas pumps contain latex should be avoided 280. The nurse is assessing a 16 month old girl. The nurse observes poor hygiene, diaper rash and boulses over the child’s body that is at different stages of healing. Which of the following interventions should reduce fear and promotes the trust of the chid? A. Avoid scaring the chid by saying “no, or setting limites B. Challenge the information the parents give regarding the injury C. Question the parents of the child over the course of hospital stay D. Assign one nurse to care for the child over the course of hospital stay 281. A patient who is prepared for hip surgery has an order for external pneumatic compression devices. The teaches the patient that pneumatic compression can help prevent: A. Upper respiratory infections B. Decrease breath sounds C. Deep vein thrombosis D. Bleeding at the surgical sites Compiled by : V.Devi | 9176868900

Page 42

Diya Coaching Centre for Nurses One candle wipes out darkness…….

282. A patient presents with a productive cough with a moderate amount of white forthy sputum and dyspnea. The patient is anxious and the nurse notices on assessment that the patient is using accessory muscle including intercostals spaces to breath and has jagular vein distension. The patient has a history of hypertension nd heart failure. What should the nurse administer first? A. Digoxin (lanoxin) to improve the ability of the heart to ump effectively B. Oxygen therapy to combat hypoxemia C. Furosemide (lasix) to reduce blood volume and pulmonary congestion D. Morphine sulphate (duramorph) to reduce anxiety 283. A nurse is preparing to meet with an individual whose spouse recently diagnosed with alzheimer’s disease. The nurse should know that the primary goals of treatment are: A. Curing the alzheimer’s disease B. Maximising the functional ability and improve quality of life C. Having the Alzheimer”s patient placed in a safe controlled environment D. Making al decision for the patient and conforming to home 284. A community nurse interviews an 87 year old patient diagnosed with Alzheimers disease. Because the patient provides conflicts information the nurse compares subjective and objective date to find a possible reason for the conflict data. The process of assessment is called: A. Data verification B. Analitical interpretation C. Mental assessment D. Subjective observation 285. The nurse assess an elderly patient for health problem. The family reports the patient has trouble remembering and they are concerned bout Alzheier’s. Why of the following are risk factors for Alzeimer’s disease? A. Genetic history and male gender B. Ethnic group and dietary habits C. Genetic history and female gender D. Dietary habits and male gender 286. A pt c Alzhemires disease has a fall which results in # of right hip following the surgical repair of the fracture the pt is discharge home c family care givers. During home visit to provide wound care the family verbalizes frustration that the pt has been Incontinent since returning home. Which of the following instructions would nurse gives to the family? A. Instruct the family in providing incontinent care and put nsg nursing diagnosis for risk for impaired skin integrity B. Suggest to the family that if the stress is overwhelming, placement in a skill nursing facility may be needed C. Suggest toileting the pt on a regular schedule and applying incontinence pt at all times D. assess for the cause of incontinence, and appropriate nsg diagnosis and intervention 287. A nurse assists apt c Alzheimer’s disese in tooth brushing. The pt indicates wanting to complete the task alone, but is a unable to get the tooth paste on the toothbrush. The nurse can most effectively help the pt by? A. Providing privacy to complete the task B. Completing task Compiled by : V.Devi | 9176868900

Page 43

Diya Coaching Centre for Nurses One candle wipes out darkness…….

C. Providing hand over hand assistance c the task D. Telling the pt to forego brushing the teeth today 288. A child c iron deficiency complains of feeling tired all the time. The nursing diagnosis of fatigue is related to? A. A decreased ability of the blood to transport O2 to the tissue B. An increased paroxysmal abdominal pain and distention to the stomach C. A decreased anxiety level during hospitalization D. decreased nutritional intake c malabsorption of nutrition 289. A patient arrives at the emergency room with burns over upper torso arm the nurse should obtain the patient’s pulse at wich of the following arterial location . Redial b. Crotid C. Femoral D. Apica 290. A patient with a spinal cord injury states “ I heve no control over my situation, I can’t do anything for myself”. This patient is exhibiting A. powerlessness B. Delusions C. Suicidal ideation D. Designation 291. nurse is teaching a prenatal class to a group of the first time mothers, each as different points in their gestation, which of the statement if TRUE regarding the management of fatigue? A. Rest flt on back, especially during the third trimester B. Exercise programs should focus on their training C. frequent 15 minutes to 30 minuets rest periods are important D. Six hours of sleep anight is adequate 292. A nurse is caring for a postoperative patient who is an subcutenious, low dose heparin. The mediation is used to prevent A. Deep vein thrombosis B. Congestive heart failure C. Paralytic ilicus D. Pneumonia 293. The pt is recovering following surgery for placement of colostomy. The nurse goes to the pt and instruct how to care for the colostomy. The pt’s roommates has visitors and pt does not want to participate at this time. What should the nurse do? A. Document the pt’s refused and non-complience on the care plan B. Tell the pt that this is vital information and may delay discharge C. Paln atime convenient to both the pt and nurse D. Pull the cartain around the bed and speak ensuring privacy 294. Which of the foowing actions would be appropriate for the nurse to take when caring for a pt on contact precaustions? A. Serve he pt’s meal on disposable plastic eating utensils B. Instruct visitors to tak to the nurse before entering the pt’s room Compiled by : V.Devi | 9176868900

Page 44

Diya Coaching Centre for Nurses One candle wipes out darkness…….

C. Rinse both hands c water after removing gloves D. Place a surgical mask on the pt during transport 295. A pt recently diagnosed c herpes zoster. The nurse establishing the care plan would most likely assign the highest priority to which of the nursing sis? A. Anxiety B. Social isolation C. Peripheral neurovascular dysfunction D. Acute pain related to disease condition 296. A nurse visit a pt at home who doesnot, understand how to take newly prescribed medicines. The prescription reds 5 ml PO TID p.c. meals. The nurse explains the patient, the correct way of taking medicine is A. 1 table spoon by mouth, three times a day before meals B. 1 teaspoon by mouth, three times a day after meals C. 1 teaspoon by mouth, three times a day before meals D. 1 tablespoon by mouth three times a day fter meals 297. In order to reduce the risk of dieseas transmission from a patient with diphtheria which of the following standard precaution who would be the nurse implemented? A. Airborne B. Contact C. Droplet D. Ventilator 298. Patient with measles(rubella is an airbone precaution, which of the foowing would be essential to implement for non immune person entering the room? A. Gloves B. Gowns C. Face shields D. Masks 299. A ptient sustained multiple musculoskeletal trauma after a motor vehicle collision and is now too skeletal traction awaiting surgery. The nurse observes that the patient has developed a large area of flat fix points purple- coloured areas on the thorax. Which of the following action would be appropriate for the nurse to do? A. Discontinue the opioid that is being administerd B. Place on extra blanket on the patient C.Reese the weights on the patient skeletal traction D. Administer diphendydrmine (Benadryl Prescribed patient allergi reaction 300. physicin has ordered gavages feeding every 4 hours for a 12 week old infat with failure to thrive is order a know how far to insert the feeding tube. The nurse should measure to distance from A. The infant’s month to the sciphoid process of the sernum B. The tip of the infant nose to the car and then to the umblicals C. The infant’s mouth to the ear and flexible amblicity D. The tip of the infant’s nose to the ear and flex to the sciphold process of the sternum

Compiled by : V.Devi | 9176868900

Page 45

Diya Coaching Centre for Nurses One candle wipes out darkness…….

301. A nurse is assessing an infant diagnosed with failure to thrive. In addition accurate anthroprometric measurement, complete nutritional history infant feeding bility, and hed to toe assessment the nurse should assess which of the following A. Parent to child interaction B. Number of sibling in the home C. Current sleep patterns D. Exposure to second hand smoke 302. n infant who weights 9kg (19.8 lbs requires 90ml fluid per day maintenance fluids the infant typically consumes 120ml during each feeding. I infant must have how many feedings per day to meet the fluid maintanence needs? A. 4 B. 10 C. 8 D. 12 303. A school nurse refers a child who failed the school vision screening for doctor. The child returns with glasses to be worn at all times. A. Redness of the eye B. Episodes of seixers C. Improved vision with glasses D. Lazy eye 304. 2 years old child in the emergency de[partment exhibits symptoms of bacte meinightis. Whixh of the following tests confirm or rule out this diagnosed? A. Magnetic resonance imaging (MRI) B. Mgnetic encephalogram C. Computed tomography scan(CT) D. Lumbar puncture 305. A patient exhibit clinical manifestation of a pulmonary embolism. Arterial blood gas (ABG) levels and a chest x-ray are ordered. Which of the following test is used to diagnose this condition? A. Computer tompgrphy scan(CT) B. Magnetic resonance imaging (MRI) C. Pulmonary function test D. Pulmonary angiography 306. A patient is admitted to the emergency department with a sucking chest wound has diminished breath sound on auscultation which of the following interventions would the nurse performed FIRST? A. Monitor O2 saturation and DG levels B. Apply petroleum gause to wound C. Prepare to patient for emergency thoracentesis D. Position the patient in an upright position 307. A patient has pulmonary embolism. Which of the following nursing diagnosis has PRORITY A. Anxiety related to pain, dyspnea and concern of illness B. Risk for injury related to altered hemodynamic status C. Acute pain related to congestion and possible lung interaction Compiled by : V.Devi | 9176868900

Page 46

Diya Coaching Centre for Nurses One candle wipes out darkness…….

D. Ineffective breathing pattern related to acute increase in alveolar dead airspace 308. Which test should be added to the yearly physical of a patient? Who has recently turned 50 years old? A. Culture and sensitivity B. Fecal occult blood C.Routine urine analysis test D. Angiography studies 309. The normal range of PH in arterial blood A.7015-7.200 B.7.25-7.3 C.7.35-7.45 D. 7.5-7.55 310. To prevent pressure on the feet of a bed bound patient with decreased tissue perfusion BEST intervension the nurse should take is; A. Place sheepskin under the feeds B. Place foot cradle on the bed C. Pad the side rails with foam tubing D. Use only natural fiber lines 311. The nurse assists with a lambr puncture on a child with suspect bacterial meaningitis. If the diagnosis is correct, the cerebrospinal fluid should have which of the following qualities? A. High glucose evel B. Low protein level C. Cloudly or turbid appearance D. Pink or blood –finged appearance 312.An elderly patient with serve degenerative joint disease comes to the clinic routine follow up of pain management. The patient reports that over the past more the pain has began to I increase in dosage of the pain mediacation. The nu recognizes that this is MOST likely due to ; A. Drug Addition B. Drug tolerance C. An improvement in condition D. Lack of efficiency of the current mediction 313.A patient had hepatitis B (HBV) and is now a chronic carrier. In plnning ... the nurse would explain an HBV carrier would most likely be at risk for developing super infection with which other type of hepatitis? a. A b. B c. C d. D 314. pre-operative patient has large volume ceansing enema ordered. In order faciitte the flow of the solution into the rectum and colon, the nurse should positive the patient in the; A. Supine position with legs flexed to chest B. Right laterl position with left sharply flexed Compiled by : V.Devi | 9176868900

Page 47

Diya Coaching Centre for Nurses One candle wipes out darkness…….

C. Supine position with legs spread D. Left ateral position with right leg sharply flexed 315. Respiratry depression is a potentially life threatening adverse effect of A. Opiods B. nticoagulants C. Immune modulators D. Non-steroidils (NSIDS 316. A child in the postictal state of seizure should show which of the following sign or symptoms? A. Feeling sleepy or exhausted B. Stiffness over entire body C. Verbalizes having an aura D. Eyes fixed in one position 317. Standards of pain management indicate that nurses A. Administer analgestic via injection whenever possible B. Avoid the use of the word plan C. Screen for pain at each encounter D. Encourage round the clock dosage of analgestics 318. The nurse observes a patient who is unable to spek when the nurse asks if the patient cn spek. The nurse observes tht the ptient is neither coughing not cyanotic. The nurse should IMMEDIATELY A. Lay the patient fat before compressinmg the midstornal area B. Insert a finger into the patient’s mouth to feel for any food C. Stand behind the patient while performing thrusts D. Activate the emergency call light near the patient 319. A patient required long-term antibiotic has a central line catheter inserted into the right subclavian vein by the physician. Which of the following must be verified prior to the first use of the catheter? A. Blood return B. X-ray C. Catheter potency D. length of catheter 320. When planning discharge teaching for a patient hospitalized for treatment of third degree burns over 30% of the body, the nurse knows it is MOST important to include which of the following instruction regarding the loss of lrge amounts of serum occurring with burns and the resulting loss of immune function? A. Wash hands frequently each day B. wear masks while in public spaces C. Wear supplement oxygen at night D. Take multiple vitamin tablet each night 321. Which of the following is the most important discharge planning instruction for a patient with mononucleosis? A. Avoid activities that may increase injury to the spleen B. Avoid crowded areas to prevent the spread of infection Compiled by : V.Devi | 9176868900

Page 48

Diya Coaching Centre for Nurses One candle wipes out darkness…….

C. Consume vitamin K rich food to decrease the risk of bleeding D. Take an antibiotic prescribed to treat infection 322. Which of the following test measures quantity of prothrombin in blood and monitors the effectiveness of warfarin sodium therapy and prolonged deficiencies in the extrinsic factor ? A. Thrombin time B. Prothrombin time C. Partial prothrombin time D. Activated partial prothrombin time 323. While conducting a class for expected mothers the nurse explains the differenced between true labour and false labour contraction by indicating that the labour contractions: A. Are located mainly in the abdomen and groin B. Have increasing intensity. C. Occur with increasing intervals. D. Occur at regular intervals 324. One month after starting new medication for hypertension pt returns clinic with blood pressure in the range. The pt admits of taking medication only when’ feeling bad’. Which of the following actions would the nurse take? A. Assess further determines the reason for pt’s action B. Add new diagnosis for non compliance C. Re- educate the pt about the importance of following his medication plan D. Re evaluate the need for daily medication since the blood pressure is acceptable 325. Home care nurse visit p[patient who is wheelchair bound due to recent motor vehicle accident. The pt has been sitting in the wheel chair for extended periods of time which resulted in the development of a stage pressure sore on the right buttocks. What is the best nursing intervention? A. Instruct the caretaker to change the position evry two hours B. Apply hydrogel to the stage 1 pressure sore every 8 hours C. Refer the pt to the wound care specialist for debridement D. Encourage the pt to consume the increased amount of calcium 326. Following n open-cholecystectomy the nurse woud instruct the patient the expect to resume normal activity. A. 1to 2 weeks B. 2 to 3 weeks C. 4 to 6 weeks D. 6 to 8 weeks 327. patient had retinal detachment surgically repaired the nurse identified the detachment would most line be correct and unlikely to reoccur if the retina remain attached at LEAST. . 3 days B. 2 weeks C. 2months D. 3 months 328. Home care nurse visits an elderly patient who had a surgical repair for fracture. The patient is taking opiod analgestics for pain.Today the patient comple of decresed appetites and absence of a bowel movement for four days which of the following can be inferred. Compiled by : V.Devi | 9176868900

Page 49

Diya Coaching Centre for Nurses One candle wipes out darkness…….

A. Constipation related to use of opioids B. Decreased appetite due to depression C. Constipation due to acute pain D. Decreased appetite due to use of opoid 329. A child recently diagnosed with sickle cell anemia is being prepared for discharge. Which of the following statement by one of the parents would require ADDITTIONL teaching by the nurse? A. high altitude can be necessary in the future B. Blood transfusion may be necessary in the future C. Strenuous physical activity should be avoided D. Increased fluid intake minimize pain 330. 13 yrs old child is hospitalized for treatment of sickle cell crisis the nurse finds the child lying on the side with the knees palled to words the abdomen. The parents are the bed side. The child is crying and does not answer the nurse the parents about the child’s pain tolerance? A. Tel the child to rest while and the nurse will return at another time for evaluation. B. Ask the child to describe the pain it is located, and to rate it on the baker pain scale. C. Ask the parents to medicate the patient with the medication ordered for break through pain as soon as possible. The resume the evaluation. D. Interview the patient about the child’s pain tolerance and usual medication requirement. 331. The nurse is entering the room of a patient who is bind. The nurse should A. Speak before touching the patient B. Talk to the patient using aloud tone of voice C. Ask then patient using can be answered “yes” or “No” D. Stand directly in front of the patient while talking. 332. patient presents to the cinic voiang concern about being exposed to hepatits (HAV) on week ago. Upon questioning the nurse finds the patients purchased food from person recently diagnosed with HAV. The nurse would be MOST correct when instructing the patient. A. The incubation period is 3 to 5 weeks B. HAV is spread by sexual transmission C. HAV is spread by blood contact D. The incubation periods is 2 to 5 weeks 333. While performing apre-operative ssessment on patient having arthoscopy of right knee, a nurse examines the right leg for baseline assessment. The nurse should include all of the following EXCEPAT A. Position and length of leg B. Bilateral pulse C. Boney prominence of ankle and feet D. Rottion of patella 334. A patient had right knee surgery and is being transferred to the post anesthesi cure unit of the following information ESSENTIAL to discuss? A. Pre operative weakness of the lower extremities B. Anxiety related to inherited risk factors of surgery C. Fear related to body image disturbance D. Allergy to aspirin based products Compiled by : V.Devi | 9176868900

Page 50

Diya Coaching Centre for Nurses One candle wipes out darkness…….

335. Patient who underwent right knee arthroplasty two days ago has nursing diagnoses of impaired mobility. The patient refuses to get out of bed ambulate due to chest pin which of the following actions would the nurse most likely implement? A. Medicate the patient prior to ambulation B. Add a nursing diagnosis of non compliance C. Let the patient rest now then try to ambulate later D. Assess to determine the cause of the chest pain 336. After total knee replacement a patient is being discharged to home after which be will ambulate four prong cane when. providing patient teaching regarding going up and down stairs with the cane the first step in going up stairs. A. Place the cane and the affected extremely up to the step B. Place the cane and the unaffected extremely up on the step C. Step-up on the affected extremely D. Step-up on the unaffected extremely 337. A nurse is caring for patient who had right mastectomy 2 days go. Which of the forming is an appropriate nursing goal for this type of surgery? A. Accept altered body image B. Avoids large croweds C. Limits right arm movement D. Perform range of motion for left arm 338. Which instruct have priority in reducing anxiety related to surgery? A. Surgical procedure and postoperative exercises B. Risk of infection after surgery C. advanced direction and what is means D. Pre-operative laboratory result and what to expect on it 339. Nurse is assessing patient who had surgery under GA. The patient’s RR is 4/mt and oxygen saturation on 3ml/mt on O2 via nasal cannula are 84%. The nurse is awaiting the results of ABG and anticipates that which of the following elevated. A. Arteril O2 saturation B. Hydrogen ion concentration C. Partial pressure of arterial O2 D. Partial pressure of arteril carbon dioxide 340. A nurse has been visiting bed-bound patient with decreased bowel mobility it the home for nurse that the patient is becoming incontinent of faces. The nurse evaluates the pan of care and notes which of the following intervention would most likely beneficial? A. An enem two times week B. increased fiber in the diet C. A routine bisacodyl (dulcolax suppository D. An enema three times a week 341. Abed bound patient has a care plan with intervensions to incude re-positioning every 2 hrs. The patient develops stage 1 pressure stage sore on the right heel. What intervention should be added to the care plan? A. Massage the right heel 4times per day Compiled by : V.Devi | 9176868900

Page 51

Diya Coaching Centre for Nurses One candle wipes out darkness…….

B. Add a trapeze to the bed C. Float heels off bed with a pillow D. Add a bed cradle to the bed 342. A patient is receiving from surgery using spinal annesthesia. The patient developes a spinal headache. Which of the following nursing actions would be most appropriate?. A. Elevate the head of the bed 30 degrees . B. Keep the patient well hydrated C. Limit intake of salty foods D. Lower the temperature of the room 343. A nurse is giving discharge planning instruction to the parents of a 1 year old child with acute ottis media. Which of the following discharge instruction taken first priority? A. Administer antibiotics as prescribed B. Breast feed as long s possible C. Administer influenza vaccination D. Avoid smoking around the child 344. Three weeks post amputation of the leg the patient is instructed to message the residual limb. The most likely rtionle for this to A. Provide counter-irritation for pain control B. Prepare for prosthesis C. Promote wound healing D. romote acceptance of the limb’s ppernce 345. A patient receives blood transfusion for severs anemia after surgery. While evaluating the patient the nurse finds that the patients oral temperature has began to rise from 98.2 0 f(36.80c to 11.00 F (38.30C. What should the nurse do? A. Give the patient an antipyretic medication and continue the transfusion an ordered B. Discontinue the IV line and restart in another site C. Stop the transfusion, keep the vein open with normal saline, and notify the doctor immediately D. Use blood cooling device to cool the blood as infuses 346. The nurse is teaching patient who has just diagnosed with bacterial conjunctivitis. The nurse should tech that the most effective way to transmission of this to other people is by A. Putting on clean gloves before cleansing the eye B. Taking medication as prescribe C. Wearing guze eye patch D. Performing hand hygiene 347. A nurse for child with celiac dises. The patient would have a permanent inability to tolerate A. Protein B. Gluten C. Fat D. Carbohydrate 348. The nurse is caring for a patient who had ligation of intracranial aneurysm via craniotomy 6 hours ago. The nurse should understand that which of the following prescribed medition will help to minimize ICP. Compiled by : V.Devi | 9176868900

Page 52

Diya Coaching Centre for Nurses One candle wipes out darkness…….

A. Morphine sulphate B. Potassium Chloride C. Warfarin sodium D. Basacody(dulcolax) 349. When planning discharge teching for the prent of an infnt with bronchiolites the nurse should empharis. A. Use of supplemental oxygen t night B. Frequent hand washing C. Sleeping in the supine position D. Rice thickened formula during night time feedings 350. A nurse is caring for a child who is post tonsillectomy and adrenoidectamy. The nurse should plan to assess which of the following complications? A. Pulmonary hypertension B. Hemorrhage C. Hearing loss D. Core Pulmonale 351. A patient has multiple clerosis and complaints of over whelming fatigue. The nurse would be most correct in instructing the patient to A. Conserve energy during activities of daily living B. Increase muscle strength through aerobic exercise C. Ignore fatigue and keep working D. Increase early afternoon intake of caffeine. 352. While caring for an edematous patient with multiple pressure sore, the nurse asked by the patient’s spouse to evaluate several menus, which of the following menus would be most therapeutic? A. Steamed carrots, milk up apple sauce B. Tuna fish with mayo naice, boiled egg and yogurt C. Grilled steak, baked potato and peach pie D. Chicken noodle soup, banana and cocoa 353. When administering an oral medication to toddler, which of the following interventions should the nurse plan to use ? A. Depress the child’s chin with thumb to open the child’s mouth B. Place the medication in a nipple for the child to suck C. Give the child a small medication cup for day D. Tell the child that the medication taste good 354. The nurse is monitoring a patient’s urine to determine hydration status. What urine colour would indicate the best hydration? A. Clear B. Amber C. Tea D. Pale gold

Compiled by : V.Devi | 9176868900

Page 53

Diya Coaching Centre for Nurses One candle wipes out darkness…….

355. A patient is being evaluated due to onset of paleness, shortness of breath and sensation of heart. Palpitations which of the following component of CBC should the nurse review to determine if the patient has anemia? A. Leukocytes B. Platelets C. Erithrocytes D. Thrombocytes 356. While a nurse is assessing a patient who reports pain due to indigestion that radiates in to the jaw. The jaw pain is rated scale of 0 to 10. The patient reports the pain started an hour ago. The nurse should immediately: A. Assess the patient’s oral temperature B. Determine what food the patient ate C. Place the patient to reverse trendelenburg position D. Obtain order and administer morphine sulphate 357. An elderly home bound patient is visited by the community health nurse. During evaluation decreased skin tuegor is noted. When asked about fluids intake, the patient states that she does not drink any fluids after lunch each day, and not wal very thirsty. The most appropriate question for the nurse to ask is : A. How much protein does you normally eats for dinner? B. How much caffeine are you consuming each day? C. Are you having trouble controlling your bladder at night? D. Do you have enough money to buy liquids/ 358. A nurse is caring for a patient who had rhinoplasty 2 weeks ago. Which of the following is an expected outcome? A. Oral mucus membranes dry, but pink and intact B. Face and nose free from swelling C. Able to make needs know. Speech therapy D. Demonstrate throat clearing while eating. 359. A patient presents to the emergency room with complaints of eye drainage planning for examination of the patients complaints, which of the following instruction would the nurse most likely select? A. Sphygmomanometer B. Thermometer C. Ophthalmoscope D. Otoscope 360. A home health nurse has completed the assessment of a 72 year old patient with a gait disturbance who will begin home physical therapy. During the interview the patient reported significant difficulty sleeping more than 4 hours at night. Which of the following responses would be appropriate for the nurse to make? A. Try doing some types of exercise 2 hours before bed time B. Drink a cup of warm tea before you go to bed C. Make sure the bedroom is dark when u get in bed D. A nap in the middle of the day should help Compiled by : V.Devi | 9176868900

Page 54

Diya Coaching Centre for Nurses One candle wipes out darkness…….

361. A nurse is caring for a 3 weeks old infant who was just admitted to the hospital. Which of the following nursing interventions does not support this infant’s basic emotional and social needs? A. Provide for continual contact between parents and infant B. Activity involve parents in caring for the infant C. Keep the infant’s environment quiet, dim and free of sensory stimulation D. Foster infant sibling relationship as appropriate 362. A homecare nurse visit a patient who is discharged from a hospital after a treatment of urosepsis. Which of the following post discharge normal laboratory results indicate best desired outcome? A. WBC count B. Hematocrit C. Platelet level D. Potassium level 363. a nurse visits a patient which is 37 weeks pregnant and asking for information about breast feeding verses feeding prepared infant formula. A beneficial reason to breast feeding includes: A. Readly available and economical B. Keep a baby full longer C. Lager curds than cows milk and therefore is easier to digest D. Encourage greater deposits of subcutaneous fat in an infant 364. When implementing a feeding schedule for a full term 2 weeks old infant, the nurse should expect the infant to be fed A. 2-4 times per day B. 6-8 times per day C. 10-12 times per day D. 14-16 times per day 365. N home care nurse makes a follow up visit to a patient who had shingles a month since the onset. The patient pain level is 6 on a scale of 1 to 10, where 1 is no pain and 10 is greatest pain. Two weeks ago the pain level was at 7. The patient is treated with pain medication as needs, and skin vesicles are completely resolved without any scaring. The patient’s condition has: A. met the expected outcome B. Partially met the expected outcome C. Has not improved D. Has worsened 366. The nurse is in public area of the health care facility when an adult falls to the floor.Which of the following actions should the nurse take next? A. Open the airway B. Determine unresponsiveness C. Activate the emergency call system D. Obtain the Automatic Electronic Defibrillator(AED 367. When caring for a patient who is receiving anticoagulant medication, the nuse must monitor the patient for signs of A. Skin breakdown B. Bleeding Compiled by : V.Devi | 9176868900

Page 55

Diya Coaching Centre for Nurses One candle wipes out darkness…….

C. Pain D. Confusion 368. A patient is being prepared for a right breast biopsy under general anesthesia. The patient asks the nurse about the surgiuacal scar and possible postoperative complications. Which of the following actions would be appropriate for the nurse to take? A. Review of postoperative risks with the patient B. Notify surgeon about the patients questions C. Complete the patient’s preoperative D. Show the patient photos of breast surgical scars 369. A patient with bowlegs due to abnormal bone formation and deformities has a calcium level of 7.5mg/100ml. Which of the following foods would the nurse most likely instruct the patient to add to a diet? A. Organ meals B. whole grains C. Egg yolks D. Lean meats 370. A patient has just diagnosed with hypothyroidism. Which of the following instructuions is correct? A. You will need to take thyroid hormone replacement therapy for your entire life B. You will need to take thyroid hormones replacement therapy until hypothyroidism C. You will need to take thyroid hormone for 2 months D. you will need to take thyroid hormone for one year 371. The stages dying as identified by Dr. Elizabeth A. Anger, depression, bargaining, denial B. Bargaining, denial, acceptance, depression C. Denial, anger, Bargaining, depression, acceptance D. Depression, denial, anger, bargaining, acceptance 372. A co-worker informs that the nurse about experiencing increased level of work associated with daily job responsibilities to help cope with professional stress, the nurse should encourage the co worker to. A. make a list of unfinished tasks B. Complete complex mental tsk before physical tsks C. Acknowledge daily accomplishments D. Spend time with colleague away from work 373. A nurse is caring for a POP patient who is on subcutaneous low dose heprin. When administering an injection on the abdomen the nurse avoids the umbilical area because of the possibility of A. Entering a large blod vessel B. Causing increased pain C. Precipitating hyper ventilation D. Umbilical infection 374. A patient with conjunctivitis reports the presence of photophobia. The nurse should teach the patient to Compiled by : V.Devi | 9176868900

Page 56

Diya Coaching Centre for Nurses One candle wipes out darkness…….

A. Avoid touching the eye B. Use sterile guaze to remove the drain C. Darken the room D. Rest in the prone position 375. During surgery the patient had following intake and output, IV fluid 650cc, IV antibiotic 50cc, 1 unit of packed RBC 350cc, NG out put 120cc, blood loss 85cc and urinein FC 240cc. What is the [patient’s total input? A. 650cc B. 700cc C. 950cc D. 1050cc 376. A nurse is performing assessment on patient’s nutritional status. Which of the following would be the best measure of patient’s recent nutritionl status. Which of the following diagnostic test would be the best mesure of the patient’s recent nutritionl status with a half life of 2-3 days? A. pre albumin B. Haemoglobin C. Albumin D. 24hr urine creatine 377. A nurse is caring for a patient with pneumonectomy 2 days ago for lung cancer. Which of the following would indicate that the patient is progressing towards discharge goals? A. Cough productive of serosangeous fluids B. 1+ pretibial edema C. Nas after completting bed bath D. Frequent ventricular contractions 378. The nurse is caring for a ptient with parkinsons disease. Which of the following is an expected outcome related to nursing diagnosis of constipation related to diminished motor function, inactivity and medications. A. The patient will use a laxative every other day B. The patient will have a soft bowel movement daily. C. The patient will report minimal pain with bowel movements. D. The patient will limit intake of complex carbohydrates. 379. A community health nurse assesses a 68 year old patient who lives in a group home The patient reports decresed apetite after transferring to the group home because the food taste too bland. What type of the data the nurse collecting from the above information? A. Analytical B. Derived C. Objective D. Subjective 380. The homecare nurse is providing wound care for a patient. The nurse evaluates the wound nd notes the presence of granulation tissue in the wound bed. This observation represents which phase of wound healing. A. Maturation B. Inflammation Compiled by : V.Devi | 9176868900

Page 57

Diya Coaching Centre for Nurses One candle wipes out darkness…….

C. Proliferation D. Finalization 381. A nurse is caring for a 3yrs old child with a fractured arm. Which of the following interventions is the most appropriate for pain management? A. Administers analgesics when necessary B. Assess pain once a shift C. Anticipate pain and intervee early D. Encourage the use of self quieting techniques 382. which statement by the patient with hyperlipedermia shows a basic understanding of the disease and its treatment? A. Exercise has no effect on cholesterol levels B. Hyper lipidemia is usually symptomatic until significant target organ dmage is done. C. HDL cholesterol level of greater than 60mg Idl increases the chance of coronary artery disease D. Chloesystyramine should be taken in the morning with other medications 383. A doctor has ordered a intramuscular injection for a 6 month old infant after her IV infiltrated. Because infant have under developed muscles, the nurse should not administrates the injection in to which muscle? A. Vastus laterals B. Rectus femoris C. Ventrogluteal D. Gluteus maximus 384. A nurse caring for a patient with cute pulmonary edema observes that the patients cough produces white frothy and that the patient is extremely dyspneic. The patient has inspirations and expiratory wheeling on auscultation of the lungs. The immediate objective of treatment is to A. Improve oxygenation B. Decrease anxiety C. Improve tissue perfusion D. Decrease risk for spirtion 385. When discussing dietary choices with a patient who is on heparin therapy the nourse should teach the patient that which of the following foods may increase clotting time? A. Grape fruit B. Orenges C. Bananas D. Red grapes 386. A 2 month old child in the emergency department has projectile vomiting after feeding. The vomiting is non bilious containing milk nd gastric juices. Immediaately after vomiting the child tries to feed again. The nurse palpates the child’s bdomen during feeding and notes a firm area to the right of the umbilicus in the ippes right quadrant. Which of the following is consistent with the history? A. Hyper tropic pyloric stenosis B. Hirchsprungs’s disese C. GERD D. Trachoesophgel fistula Compiled by : V.Devi | 9176868900

Page 58

Diya Coaching Centre for Nurses One candle wipes out darkness…….

387. A patient undergoing cancer treatment has developed acute hyper calcemis with signs of weatness, nausea and vomiting. Which of the following would the nurse anticipate to be the initial treatment? A. Thiazide diuretic B. Intrvenous normal sline(0.9% NACL) C. A pttasium supplement D. Broad-spectrum antibiotic 388. The patient is receiving mechanical ventilation set at fraction of inspired oxygen (FI02) 100%. The nurse should understand that which of the following can improve the patient’s oxygenation? A. Adding positive end expiratory pressure(PEEP B. Placing the patient in Trendlenbueg position C. Increasing the FI 02 D. Suctioning the patient hourly 389. Which of the following nursing diagnosis takes priority for a patient after gastrointestinal surgery? A. Impared skin integrity related to surgical incision B. Constipullation related to surgery C. Risk for infection related to surgical incision D. Acute pain related to surgical incision 390. Marijuana is an example of a drug classified as schedule A. C I B. C II C. C III D. C-IV 391. A patient with a weight loss of 12 in 60 days has a nursing care plan written interventions including offering a dietery supplement three times per day> After 2 weeks, the patient has had another 1% weight loss. The patient indicates no likely the supplements. The nurse should A. Continue the pln of care as written B. Replace the supplement with high calorie food the the patient likes C. Encourage the patient to drink supplements D. Offer smaller amounts of supplement more frequently 392. The nurse is caring for a patient with magnesium toxicity. Which of the following clinical manifestation should the nurse anticipate? A. Parasthesia B. Decreased deep tendon reflexes C. Cardiac palpitations D. Decreased cardiac output 393. A ptient returning from 3 hours shoulder repair with general anesthesia is being transported from the operating room (OR) to the post anesthesia care unit(ACU). The nurse knows that the patient is at high risk for injury related to residual annesthesia. During this timeperiod the patient is at lwest risk for A. Airway obstruction Compiled by : V.Devi | 9176868900

Page 59

Diya Coaching Centre for Nurses One candle wipes out darkness…….

B. Vomiting C. Impared circulation D. Fluid volume deficit 394. For a patient schedule fr a total pancreatomy the nurse would be instruct the patient that hte procedure most likely cause A. Pancreatic ascites B. Chronic pancreatitis C. Diabetics mellitus D. Diabetics incipedus 395. The nurse is assessing an infant for possible deafness. Which of the following automatic reflexes would the nurse most likely check to best determine whether the child has a serious hearing problem? A. Blinking B. Vertical suspension C. Moro D. Perez 396. The nurse is teaching a group about aerobics exercise. When discussing the trget heart rate for exercise, the nurse should state that this is calculated by A. Counting the number of the heart beats during exercise for 6 sections, then multiply this number by10 B. Subtracting the chronological age from the number 220 C. Counting the number of heart beats during exercise for 10 seconds, then multiply by 6 D. Subtracting chronological age from 240 397. While performing an assessment on a post surgical patient 2 days after surgery, the nurse notes shallow and rapid respirations. What should the nurse do next? A. Assss the patient for pain B. Obtaian an order for supplementary O2 C. Elevate the head of the bed D. Place a warmed blanket on the atient 398. A patient is receiving intravenous fluids of a ration of 125ml/hour. What vlume fluids will the patient receive during an 8 hours shift? A. 1500ml B. 1 litter C. 1.5 litter D. 500ml 399. A patient has a history of severe, uncontrolled epistaxis. The patients blood pressure and patient count are normal. The nurse should teach the patient to: A. Sleep with the head elevated on at least two three pillows B. Apply firm pressure to the nostrils four times a day C. Use a cotton tipped applicator t apply a water soluble lubricant to the nasal spectrum twice daily D. Minimise the intake of caffeine while using the intake of fluids rich i vitamin K 400. A nurse is caring for an infant with respiratory distress syndrome. Which of the following nursing interventions is appropriate? Compiled by : V.Devi | 9176868900

Page 60

Diya Coaching Centre for Nurses One candle wipes out darkness…….

A. Measure oxygen saturation level once a shift B. Suction frequently for 30-45 second each time C. Monitor for symptoms of hyper glycemia D. Maintain infant temperature b/n 367 degree and 37.8 degree celcious (970 and 980 F) 401. A nurse is caring a patient who had a left mastectomy with lymph node removal seven days ago. The patient asks about exercises to regain function of the left arm. Which of the following activities would be most appropriate? A. Walking fingers up the wall B. Using five pound weights C. Rhythmic clapping D. knitting with large needle 402. What occurs during cadiogenic shock and results in inadequate tissue perfusion? A. increased resistance of arterial vessels B. Decreased effectiveness of the heart as pump C. Increased shunting of critical blood flow to heart D. Decreased capacity of the venous beds 403. The nurse is caring for child admitted with viral pneumonia. Which of the following nursing diagnoses should receive priority? A. Nutrition altered less than body requirements B. Ineffective airway clearance C. Fluid volume deficit D. Risk of injury 404. A child has injested an entire bottled acetaminophen(Tyenol). Which of the following organs is affected? A. Liver B. Brain C. Kidneys D. Gallbladder 405. A home care nurse make a follow up vist to a patient who recently suffered a cerebro vascular accedent. The patient is mobile and able to perform activities of daily living. However, the patient has not been sleeping and has lost weight due to lack of apetite. The patient also feels overwhelmed with sadness. Which of the following is the most appropriate evaluation? A.. Patients progress is unexpected and no further intervention is necessary B. Patient needs referral to a nutritionist. C. Patient needs intervention for depression D. patient needs sleeping medication 406. A patient is seen in the emergency department with complaints of angina. Nitroglycerin (Nitrostat) is ordered by the physician. This medication is to be administered via which of the following routes? A. Intrademal B. Buccal C. Parental D. Topical Compiled by : V.Devi | 9176868900

Page 61

Diya Coaching Centre for Nurses One candle wipes out darkness…….

407. The nursing is teaching a group of patient about hepatitis A (HAV). The nurse should state that HAV is mainly transmitted via A. Blood contact B. Food C. Sexual activity D. Salvia 408. A child was recently diagnosed with spastic cerebral palsy. Which of the following statements by the parent would indicate to the nurse that parent understand teaching about this illness? A. Full recovery is possible B. This illness should not progress C.cerebral palsy is a hereditary disease D. Surgery cn some times improve walking 409. patient hospitalized with crohn’s disease has developed fever, increased respiratory rate increased heart rate, chills, diaphoreses, and used abdominal discomfort. The nurse knows that patient has most likely developed A. Intestinal obstruction B. Intestinal parasite infection C. intestinal perforation D. Asitis 410. A child is admitted to the hospital with dehydration. The nurse should give priority to which of the following nursing dignoses? A. Anxiety related to hospitalization B. Fluid volume deficit related to vomiting C. Imbalance nutrition less than body requirements related diarrhea D. Risk for infection related to presence of invasive lines 411. The nurse is caring for patient with DVT. The patient’s heparin sodium infusion has been discontinued and the patient is receiving prescribed warmfarin sodium (Coumadin. The nurse advice the patient that which of the following needs to be continued? A. Daily CBC B. Laboratory test for partial thromboplastin time (PPT C. Strict bed rest D. warming elasticized support stockings 412. hen teaching the parents of neonate with spina fibida tequniques to promote bladder emptying the nurse reviews atechnique in which firm gentle pressure is applied to the abdomen pressed towards the symphysis pubis. This method is known as A. Credes B. Intermittent C. Foley D. prophylactic 413. A 50 years old patient is being admitted to the hospital in a vegetative stte of unknown etiology what is the priority nursing diagnosis? A. Risk for impaired skin integrity B. Impired swallowing Compiled by : V.Devi | 9176868900

Page 62

Diya Coaching Centre for Nurses One candle wipes out darkness…….

C. Altered cerebral tissue perfusion D. Altered thought processes 414. Prior to administrating an enema, the nurse will assist the patient to assume what position A. Prone with pillow under the knwws B. Left side with right knee flexed C. Right side with left knee flexed D. On back with head of bed flat 415. A nurse interviews a patient, recently admitted to long term care facility , to obtain information on the ptient’s health perception. The nurse encourage the effective in this situations? A.Analytical B. Focused C. Closed D. Open-ended 416. When selecting activities to help develop child’s fine motor skills, which of the following would best meet this goal? A. Sorting cardboard objects that are in different shapes B. Singing while turning the pages of a book that plays music C. jumping rope D. Riding a three-wheeled bicycle 417. 6 years age a patient weighed 73 kg (161 pounds). During the current clinic visit the nurse note the patient has an unintended wt loss. This wt loss over 6 months would be considered clinically significant as soon as it reaches the point of being more than a A. 5%loss B.8%loss C.10%loss D.20%loss 418. child with a diagnosis of tetralogy of fallot is scheduled to be discharged from the hospital the nurse planning discharge education should instruct the care gives that during a hypercyanotic spell the position most likely to benefit the child is A. Supine B. Side-lying C. Prone D. Knee-chest 419. A child is treated for possible acetaminophen (Tylenol) overdose. The child is currently stable with normal vital signs. Which of the following organ function system would be most affected A. Liver B. Stomach C. Lungs D. Heart 420. The nurse is caring for a patient with stage III pressure ulcer to the coccyx. There days after initialling the plan of case, the nurse observes that the ulcer has hard black crust covering the center of the ulcer. The nurse should understood that this indicates A. healing Compiled by : V.Devi | 9176868900

Page 63

Diya Coaching Centre for Nurses One candle wipes out darkness…….

B. Need for debridement C. Inadequate nutrition D. Infection 421. To limit drug interactions, the nurse should advise the parent of chronicallt ill child to A. Refers to the medictions by the generic name B. Teach the child the name of all medications prescribed C. Give all medications one hour apart D. Get all prescriptiuons filled at the same pharmacy 422. The nurse receives an order to obtain an artetial gas (ABG specimen on a patient. The nurse will use the redial artery to obtain the specimen. Which of the following will the nurse assess before puncturing the redial artery? A. llen test B. Prtial pressure of rterial oxygen C. Partial crbon dioxide D. Prothrombin time 423.For an infant with hydrocephalus, anurse should plan to monitor for what sign or symptom of increased intracranial pressure? A. High pitched shrill cry B. decrease in systolic blood pressure C. Depressed frontanelle D. Increase in respiration 424. During a surgery requiring general anestasia, the patient’s heart stops. Ventilations using the endotracheal tube (ETT are started with an ambu bag. Which of the following compression to ventilation rates is correct? A. 1 to 2 B. 15 to 2 C. 5 to 2 D. 1 to 2 425. The nurse is caring for apatient scheduled for left arm amputation due to bone crcinom. Dequate ssessment and management of preoperative pain will result in A. Decreased phntom limb sensation B. Increased ROM after surgery C. Decresed depression after surgery D. Decreased likelihood of cancer recurrence 426. A 34 years old quadriplegia patient resides at home with his wife. In order to prevent contractures of all extrimities, the community care nurse will instruct the patient’s wife in the performance of A. Active ROM exercises B. Passive ROM exercises C. Active assistive ROM exercises D. Resistive ROM exercises 427. A 7 years old child is briught to the emergency room with complaints of the sick for 3 weeks with sore throat, cough and muscle pain. Upon examination, the nurse notes alow-grade fever, Compiled by : V.Devi | 9176868900

Page 64

Diya Coaching Centre for Nurses One candle wipes out darkness…….

shortness of breth, and a wheeze on uscultation child lives with prents, 6 siblings and grandfather in a 3 bedroom house. Based these findings, which of the following diagnosis would be most likely A. Staphylococal pneumonia B. Pneumocystis crinji pneumoni (P C. Bronchiolities D. Mycolsm pneumonia 428. A patient comes to the emergency department complaining of severe crush substernal pain that radiates to the left arm & jaw. The patient is diaphoretic and with cool clammy skin. The patient is diagnosed with acute myocardial infarction nursing diagnoses would be decreased cardiac output related to A. Structurl factors (Incompetent valves B. Impaired ventricular expans C. Impaired contractility D. Fluid volume deficit 429. After a hearing restoration operation, a patient has no signs of complication soon recovers which of the following is an executed outcome 5 days after the restoration surgery? A. Regain full hearing B. Minimal facial nerve paralysis C. minimal urinary incontinence D. Ambulate without difficulty 430.When teaching apatient how to use a cane after a CVA. The nurse should sure the patient A. Uses the cane on the unaffected side B. Advances the cane simultaneously with affected limb C. Holds the cane away from the body D. moves the cane past the toes of the affected limb 431. A home care nurse visits a patient diagnosed with diabetes mellitus whose current glucose level rages from 15mgldl to 200mgldl. The patient has not be to self administration prescribed bed insulin and complains of blurred vision and a inability to read the marking on the proper insulin dosage. Which of following referrals would be most beneficial to the patient? A. A dietician B. An endocrinologist C. An ophthalmogist D. A physical therapist 432. The doctor has orders the patient to be on 1 to 3 liters of oxygen using a canula at all times. The home care nurse notes the O2 is currently at 2 as/min O2 saturation SO2 residing is currently 85% and the partial pressure (Paco2 I normal limits. Based on an evaluation of this information, which of the follow actions would the nurse most likely perform? A. Decrease the O2 to IL/min & monitors O2 saturation B. Continue the O2 t 2L/min & monitors O2 saturation C. Increase the O2 to 3 L/min & monitor O2 saturation D. Continue the monitor O2 saturation & call the doctor for new order 433. A child is treted for bacterial meningtits with an intravenous antimicro agent. Which of the following Best indicates effectiveness of the treatment? Compiled by : V.Devi | 9176868900

Page 65

Diya Coaching Centre for Nurses One candle wipes out darkness…….

A. Incresed appetite B. Temperture 37.2 degree celcious (99 C. Episodes of apnea D. increased icp 434. A patient with (GERD) is to start taking prescribed omeprazole (prilsec. The nurse would instruct that patient to take the mediction A. 30 to minutes before meals B. 90 to 12 minutes before mels C. with apple sauce D. with milk 435. A patient recently diagnosed with multiple sclerosis has been taking the following prescribed medications beclofen (Lioresal) diazepam(valium Amantadine(symmetrel and phenytoin (dilantin) when the patient presents with complaint of ftigue, the nurse should address the dosage and frequency of which medication? A. Baclofen (Lioraseal B. Diazepam (valium) C. Amantadine (symmetrel) D. Phenytoin (Dilantin 436. A patient is receiving ablood transfusion post-surgery. The nurse is abut to take the frst set of 15 minutes vital signs when the patient states “I am freezing cold and my chest feels tight what should the nurse do next? A. Stop the transfusion, keep the vein pen with normal saline and notify the ordering physician B. Continue the transfusion, start normal saline, and re check vital signs in 1 minute C. Slow the transfusion to hlf the rate and administer acetaminophen (tylenon) per standing order D. Stop the transfusion, bring the patient ablanket and notify the physician 437. nurse is caring for a patient who is receiving from radio frequency abletion of matastic liver tumors. The patient has been laughing and visiting with friends throught out he shift. Shortly before the end of the nurse’s shift, the patient uses the call bell to request medication. The patient rates the pin as a 9 on scale of (no pain) to 10(severe pain). The nurse observes that the patient is smiling and respirations are evenand unlabored. Thenurse should A. Wait before administering the pain medication B. Administers pain medication as ordered and assess for effectiveness C. Review the patient’s chart for any history of drug abuse D. remind the patient to request analgesic before the pain is severe 438. A patient is being treated for asthma in the emergency department. An ABG is ordered to evaluate the patient’s oxygenetaion prior to treatment. What is the name of the procedure shown in the graphics A. Allen test B. Arterial puncture C. papanicolaou test D. angiography

Compiled by : V.Devi | 9176868900

Page 66

Diya Coaching Centre for Nurses One candle wipes out darkness…….

439. A 51 yrs old pt in hospital clinic is scheduled for colon biopsy. The pt speaks a different language than the hospital staff, but does understand simple communical in the language of staff. When conducting pt education prior to the procedure the nurse should plan? A. Write all communication and avoid speech B. Raise the volume and pitch of the voice C. obtain an interpreter D. smile and nod frequentlty 440.The following medictions are ordered for a pt who had a rigid leg amputation oxycodone sing every 4 hrs as needs and morphine 5 mg every 4 hrs as needed. The nurse dministered oxycodone 2 hrs ago but the pt report pain rated 8 on a scale of (no pain) to 10 (serve pin as the dressing change begins. Vital signs are Bp-169/98, HR-112, RR-22, T-36.7 C. After evaluating the effectiveness, of the pain medication What ct ion should the nurse take? A.Administer is additional oxycodone 5 mg B. Administer morphine 5mg C. Change the dressing quickly D. Encourage deep breathing 441. child is dmitted with temperature of 38.5 degree celkcious(103.3 degree F loss of appetite and the nurse observe several joints and red, swollen, warm and render tc touch. Non pruritic rash is on the child’s trunk laboratory test results include on ESR, a positive proteien nd n elevated WBC. The nurse should initiate the plan of care for A. CHF B. Meningitis C. Rotovirus D. cute rheumatic fever 442.A nurse is assessing the pheripherl circulation of the patient’s extremities. The chart indicates the patient has edema in both lower extrimities. Which of the following assessment tequenics would be the nurse likely use to assess for this? A. inspection and auscultation B. Inspection nd papation C. palpation and percussiod D. Percussion and auscultation 443. a child is admitted with a temp of 38.5 c loss of appetite and malaise. The nurse observes severl joints are red, swollen warm and render. A nonprurritic rashnis on the childs trunk. Laboratory result shows elevated ESR, +c C reactive protein and an elevated WBC. The nurse should initiate the plan of care for. A. CHF B. Meninggitis C. Rotavirus D. Acute rheumatic fever 444. nurse is carring for a hospitalized diabetic patient with advanced peripheral neuropthy. Which of the following nursing ction is the most important? A. moisturizing the skin with lotion ech day B. Ensuring that foods are not too hot Compiled by : V.Devi | 9176868900

Page 67

Diya Coaching Centre for Nurses One candle wipes out darkness…….

C. facing the patient when speaking D. Assessing the heels for breakdown 445. A patient in a long term care facility is in persistent vegetative state with a high contracture of right arm and hand. What is the best goal over the next 90 days for this patient related to nursing diagnosis of impaired mobility? A. Develop no further contractures B. wear an arm and hand splint C.Have no pain related to contracture D. Have the contractures resolved 446. A postoperative patient has the nursing diagnosis of ineffective airway clearance. To assess for airway clearance the nurse should check all the following except. A. Heart rate B. Skin colour C. Respiratory rate D. Breath sounds 447. the ability of the test to correctly identify those individuals who have disease is known as A. Specificity B. Sensitivity C. Incidence D. Predechive value 448. The facilitate self-care for a 2 years old child with spastic cerebral palsy the nurse should recommend A. Placing straws in to beverage containers B. Obtaining eating utensils that have large handles C. Purchasing shoes that have an open heels area D. replacing zippers on clothing with mental snaps. 449. A 21 years old female is being discharged after 2 day admission for PID. Hich statement best identifies the patients understanding of follow-up care for PID? A. My sexual partner needs to be treated with antibiotics B. Its OK to resume sexual relation now C. I need to inform any sexual partners. I have had in the past 3 days that I had PID D. In order to prevent getting PID I need to continue to take birth control pills 450. A health 2 years old child is brought to the community health clinic for routine check up. At this visit the nurse should administer the following vaccine A. Rot virus B. Hepatitis B C. None of this time D. Varicella 451. During an evaluation at a community clinic the patient completes the medical history. Which of the follow is NOT risk factor for an cute MI? A. Coronary artery disease B. smoking C. Hemophilia Compiled by : V.Devi | 9176868900

Page 68

Diya Coaching Centre for Nurses One candle wipes out darkness…….

D. Hyperlipidemia 452. Which of the following is the most common type of crdiomyopathy in children and is treated with medications such as digoxin (Lbixin nd wrfarin (Coumadin) A. Hypertrophic B.Dilated C. Restrictive D. Distolic 453. The responsibility for teaching patient how to take medications safetly when they are discharged from the hospital belongs to the A. Physician B. Detitian C. Therapist D. Nurse 454. A nurse is discharging a patient after hospitalization due to myocarditis. Which of the following statements should be included in discharge teaching? A. There is usually some residual hart enlargement B. May resume previous activities as before hospitalization C. Avoid immunizations against infections disease D. Rapidly beating hart is a common side effect f the illness and is not dangerous 455. A nurse is assessing a 4 month old formula fed infant . The parent repots the infnt has been irritable, crying excessively, not sleeping well & vomiting GERD is expected. What nursing intervention should the nurse expect to teach the parent? A. Place the infnt in an infant seat after eating B. Give large frequent feedings C. position the child in a swing D.Thin formul with wters 456. An adult arrived at the outpatient facility due to the onset of chest pain the patient suddenly falls to the floor and is unresponsive . What action should the nurse take next? A. Active emergency call system B. open the patient’s airway C. Check for croid pulse D. administer 2 rescue breaths 457. A patient suffered a head truma which resulted in a nasal fracture requiring surgical intervention.Which of the following nursing diagnoses would most likely be a problem this patient? A. Delyed surgical recovery B. Impired gas exchange system C. Ineffective breathing pattern D. Risk for periopertive positioning injury 458. After administering inhaled corticosteroids to hospitalized child with asthma, the nurse plans to have the child rinse the mouth nd gargle with water. The nurse knows the rationale for this action is prevension of A.Tooth decay B. Oral candidiasis Compiled by : V.Devi | 9176868900

Page 69

Diya Coaching Centre for Nurses One candle wipes out darkness…….

C. Dehydration D. Hypertrophy of the gums 459. The nurse is assessing a patient who is 2 weeks postoperative a kyphoplasty of 12 & L3. The patient has been partcicipating in physical therapy and has been doing daily stretching and strengtherning. Which of the following would A. reports pain in legs while sitting B. Urinating every 2 hrs while awake C. Fatigue after performing activities of daily living D. Ambulates outdoors without assistive device 460. When a child is brought to the emergency department with acute epiglottis, which of the following nursing diagnosis should receive priority A. Ineffective airway clearance B. activity intolerance C. Fluid volume deficit D. Impaired verbal communication 461. The nurse is reviewing the medication of a patient who is scheduled for a CABG in 3 days. Which of the following medictions must be discontinued t least week prior to surgery? A. Digorcin (Lanoxin B. Furdsemide (Lasix) C. Propanalol Hydrochloride (Indeeal) D. Warfarin Sodium (Coumadin) 462. A patient with pneumonia experience ineffective airway clearance related to the presence of secretions secondary to infection: O2 saturation is 89% on room air. Which of the following nursing interventions takes priority? A. Deliver O2 with humidity B. Encourage fluid intake C. assist patient in to position of comfort D. Inspect patient in to position of comfort 463. A nurse is assessing a 5 month old infant. The parents state that infant is irritable, crying excessively, vomiting formula (not projectile), acting and stiffening. Based on this assessment, what diagnosis should the nurse anticipate? A. Esophagel atresia epith tracheoesopha geal firstula B. Gastro esophageal reflux C. Hirschsprung’s disease D. Celiac disease 464. A patientpresents at the clinic with weight loss and complaints of troble seeing at night. The nurse also observes numerous teeth with decay. Upon leading that the patient has a vitamin deficiency. Which of the following foods would the nurse likely instruct the patient to ass to diet? A. Cheese and breads B. Liver and milk C. Fish and rice D. Fruits and vegetables Compiled by : V.Devi | 9176868900

Page 70

Diya Coaching Centre for Nurses One candle wipes out darkness…….

465. While providing discharge teaching for the parents of a child newly diagnosed with cystic fibrosis the nurse includes teaching regarding the role of salt in the disease. Which of the following statements by the parent indicates the need for further teachings? A. salty foods may be eaten on occasion B. My child does not need to restrict salt intake C. salt is lost more rapidly in hot weather D. salt replacement should occurs every day 466. A patient visiting the clinic `1 days after sinus surgery for check up, complies of bad taste in the month. When the nurse smells foul odor while examining the patient mouth the nurse suspects the patient have A. pulmonary decompensation B. Hemorrhage C. Aspiration D. Infection 467. A patient is scheduled for a total hip arthroplasty. The preoperative nurse review the chart and notes the following serum potassium level of 2.8mfgfq/1AB positive blood type, and elevated ST segments on the ECG. Which of the following would be the most appropriate action for the nurse to do next? A. report abnormal diagnosis results to the surgeon B. Review the patient consent for the surgical procedure C. Educate the patient on the risk factors and side-effects D. Ensure that the patient has a post-surgery physical therapy order 468.Which of the following discharging planning instructions takes priority in patient with congestive heart failure A. Maintaining a low cholesterol, low sodium and low potassium diet B. Recognizing signs & symptoms that requires immediate medical attention C. The importance of remaining physically active D. The importance of drinking plenty of fluid 469. Following ocular surgery the nurse establishes care interventions to include orienting the patients to new changes in environment and supervising the patient’s ability to feed themselves and perform self-care activities. Which of the following nursing diagnosis do these activities support? A. Activity intolerance B. Impaired envioirnmetal inter[retation syndrome C. Disturbed sensory perception D. Risk for automatic dysreflex 470. During the immediate ostoperative period, a patient reveals an O2 saturation level of 91% The nurse should A. Position the patient on the left side B. Administer supplemetal oxygen C. Continu to provide supportive care D. Lower the temperature of the room 471. Which of the following goal take priority when recovering from GA in post anesthesia care unit? Compiled by : V.Devi | 9176868900

Page 71

Diya Coaching Centre for Nurses One candle wipes out darkness…….

A. Thermo regulation B. elstic skin turgor C. Patient airwy D. patient voids freely 472. A patient is to receive heparin sodium 500 unit subscutaneous on call to the O2 prior to administering this medication the nurse advanced patient that this will help prevent A. Infection B. Atelectasisc C. thrombosis formation D. Positioning injuries 473. When administering an IM injection to an infant which of the following site is appropriate for the nurse to use? A. rectus femorius B. Deltoid C. Dorso gluteal D. ventro gluteal 474. A ptient is admitted to the medical unit with a diagnosis of fluid volume de and hypotension. Which fluid would the nurse expect the doctor to order? A. .9% sodium chloride B. .45% sodium chloride C. Dextran in NS D. 5 % sodium chloride 475. The nurse is discussing HIV with a group of high risk patients. The nurse should state that this virus is found most commonly in which of the following body fluid? A. Blood B. Saliva C. Breast milk D. Vaginal secretion 476. A parent is concerned their year old child has 203 kg (5 lb) over the past 2 weeks and has been urination up to 30 times per day. The child also seems to be eating and drinking constantly. Which test would be most helpful in evaluating the child’s condition? A. Chest X-ray B. Complete blood count C. Body fat analyisis D. Bood glucose level 477. A patient has been transferred to the medical unit following a parathyroidectomy. Surgery was performed under general anesthesia and the patient’s diet may advance as tolerated. The patient requests a sip of apple juice. The nurse should first assess the patient’s ? A. Skin turgor B. Cough reflex C. Lung sounds D. Bowel sounds Compiled by : V.Devi | 9176868900

Page 72

Diya Coaching Centre for Nurses One candle wipes out darkness…….

478. The nurse sustains a needle stick injury after administering an intramuscular injection to a patient. It is recommended that the nurse be tested for human immune deficiency virus (HIV)? A. Immediately with repeat test in 6 weeks B. If the patient refuses HIV testing C. If the patient has symptoms of HIV infections D. A month after taking prophylactic anti virals 479. A parent brings their teenage child to the paediatric office. The parent reports that the patient frequently complains of abdominal bloating and stomach pain after eating and also has a chronic sore throat. The patient’s lab shows hypokalemia. Which of the following diagnosis should the nurse anticipate? A. Anorexia nervosa B. Bulimia C. Morbid obesity D. Impulsive behaviour 480. A Urine analisis is best evaluated for accurate results if specimen is analyzed within? A. 1 hour of collection or refrigerated until analised B. 1 hour of collection or eft at room temperature C. 2 hour of collection D. 4 hour of collection 481. The nurse has started intravenous fluid therapy on a child. Which of the following action is appropriate? A. Using paddedarm board only if the child is active B. Checking the site atleast once every 2 hrs C. Determining the total volume infused every 4 hour D. Using an infusion pump to provide controlled rate of infusion 482. During the assessment phase of a preoperative interview, the patient reports feeing nervous. The patient conveys to the nurse that a parent died in surgery due to malignant hyperthermia. To whom the information be most pertinent? A. PACU nurse B. Scrub nurse C. Anesthetic team D. Charge nurse 483. A child present to the emergency department with difficulty breathing. The childs parents reports that child has history of bronchial asthma and has recently had a upper respiratory tract infection (URI). Upon auscultation, the nurse found decresed breath sounds in the left ower lung field. The nurse should Next assess the child’s A. Oral temperature B. O2 saturation C. Apical pulse D. Level of comforts 484. A patient with diabetic retinopathy is experiencing n episode of unresolved hemorrhage in the eyes. The nurse identifies the most likely procedure to benefit this patient would be A. Enuceation Compiled by : V.Devi | 9176868900

Page 73

Diya Coaching Centre for Nurses One candle wipes out darkness…….

B. Radial Keratotomy C. Viterectomy D. Peripheral iridectomy 485. A Patient admitted t the hospital with acute cholecystitis, is scheduled for surgery in the morning, and is NPO. At 2 pm the patient develops fever of 102.4F (39.1 0 C). Medication orders include acetaminophen 650mg orally every 4 hours as needed. The nurse should? A. Give the medication as ordered by the physician B. Administer the ordered dose rectally C. Put moist cool cloth on the patients forehead and axilae D. Notify the physician and request other orders 486. A patient is 3 weeks post operative left the below amputation. Which of the following is an expected outcome for this patient? A. Verbalizes relief of incision pain has intense phantom sensations B. Participates in care plan, express concern about independence C. Full passive range of motion, requires assistance with transfer D. Low grade temperature, dressing reinforced every hour 487. During post operative neurovascular assessment of a patient who had a total knee amputation, nurse assess the peronneal nerve by testing sensation? A. On The bottom of the foot B. In the space between the great and second toe C. In the area anterior t rectum D. In the anterior portion of the calf 488. The nurse is caring for a patient who is sustained atraumatic brain injury 4 days ago. The patient remains In a pharmacologic induced comma while receiving mechanical ventilation. The patient is on NPO status and the vital signs are within the normal range. The patient bowel sounds are absent and nasogastric tube is connected to low intermittent suction. The nurse should prepare to begin? A. NG feeding B. Rapid weaning from ventilator C. Total parental nutrition D. Chest physiotherapy 489. The nurse preparing to assess patients. Which of the following actions by the nurse will help to ensure the vital signs are accurate? A. positioning the patient supine before checking the temperature with tympanic thermometer. B. Placing the middle index finger over the radial artery to assess the pulse rate. C. Positioning the hand on the chest of an infant to count the respiratory rate. D. Placing the patient’s elbow on a support with the palm upward before checking blood pressure 490. A 12 year old patient had a cast removed from the left leg after wearing it for 3 weeks. The patient wants to resume sports as soon as possible. Inorder to regain the muscle strength lost while wearing the cast , the nurse will instruct the patient to perform A. Resistive ROM Compiled by : V.Devi | 9176868900

Page 74

Diya Coaching Centre for Nurses One candle wipes out darkness…….

B. Passive ROM C. Active assistive ROM D. Active ROM 491. During intra operative period of surgical procedure a 39 year old male has the following vital signs. T- 98.6 F, HR -62, BP -132/78 mmhg, SP o2- 89% . The patient has received 1 Unit of PRBC and he is in incubator. Which of the vital signs considered out of normal range? A. Temperature B. Oxygen saturation C. BP D. HR 492. A 28 year old male is recovering from moderate concussion following a motor vehicle accident 2 weeks ago. When he suddenly develops increased thrust thriving for cold water> The patient urinates very large amount of dilute water like urine with specific gravity of 1.001-1.005. The patient is MOST likely developing? A. Diabetis mellitus B. Diabetis incipidus C. Hypothyroidism D. Thyroid storm 493. A nurse is caring for a patient who is 6 hours post left lobectomy. On assessment the nurse observes that the patient is becoming very restless And nailbeds are blue and the vital signs reveals tachycardia, tachypnoea and the blood pressure is rising. Which of the following cause is most likely? A. Pneumonia B. Hypoxia C. Post operative bleeding D. Bronco pleural fistula 494. A nurse is assessing a child with cystic fibrosis. After thoroughly assessing respiratory status the nurse should assess which of the following? a. The level of p[ain B. Skin turgour C. Clarity of urine D. Nutrition status 495. A patient with heart failure has the following vital signs, BP-136/84mm of hg, HR-48/min, T98.6 F, RR-20/min. Which of these vital signs should be reported to the physician prior to administer the next dose of Digoxin? A. Heart rate B. Blood pressure C. Temperature D. Respiratory rate 496. A nurse is caring a patient 2 hour after pacemaker placement. The patient suddenly start complaining of chest pain and the nurse observes dyspnoea, cyanosis and absent breath sounds on left side> Which of the following complication is anticipated? A. Haemothorax Compiled by : V.Devi | 9176868900

Page 75

Diya Coaching Centre for Nurses One candle wipes out darkness…….

B. Perforation of heart C. Pneumothorax D. Hemorrhage 497. A nurse is preparing to administer 100m of KCl solution. The prescription indicates that this should be infused over 2 hours. The nurse should administer howmany ml per hour? A. 10 B. 25 C. 50 D. 100 498. While caring for a 5 year old patient with cystic fibrosis, the nurse enters the room to administer a prescribed abbuterol nebulizer treatment. The patient is restless and anxious and the lips have a blue gray colour. The O2 saturation is 90% what should the nurse do first? A. Administer the nebuizer treatment as ordered B. Administer oxygen C. Prepare to administer CPR D. Suction the airway 499. A community health nurse instructing a neighbourhood class about botulism (Clostridium Botulism). The nurse teaches the group that the Most likely mode of infection would be by? A. Direct contact with contaminated soil B. Direct mcontact with respiratory secretions C. Sexual intercourse D. Injestion of contaminated food 500. A patient presents to the office for a physical. The patient is found tobe health and fit, but occasionally drinks alcohol and has unprotected sex. What is the best nursing diagnosis/ A. Health seeking behaviour B. Knowledge deficit high risk behaviour C. Low sef esteem D. Altered through processes 501. A home health nurse is preparing to administer a subcutaneous injection of heparin. When choosing the site on the abdomen, the nurse will choose a site? A. More than 6 inches from the umbilicus B. More than 2 inches from the umbilicus C. Within 1 inch of the umbilicus D. As close as possible to umbilicus 502. A patient with pulmonary emboli complains of pain, dyspnea and a fear of dying. Which of the following interventions would Most likely help to reduce the patients anxiety level? A. Administer oxygen as ordered B. Administer pain medications as ordered C. Observe closely for signs of pain and discomfort D. Listen to the patients concern 503. A patient with bacterial meningitis is treated with intravenous antimicrobial agent. Which of the following Best indicates effective treatment? A. Severe headache Compiled by : V.Devi | 9176868900

Page 76

Diya Coaching Centre for Nurses One candle wipes out darkness…….

B. Negative kerning’s sign C. Nuchal rigidity D. Photophobia 504. While caring for a patient with Pottacium deficiency the nurse would expect that the patient may exibit? A. Dysrhythmis B. Olyguria C. Diminished deep-tendon reflexes D. Hypertension 505. A patient who underwent hand surgery requiring general anesthisia presents to the post anesthesia care unit (PACU). After extubation the nurse should first assess? A. Circulatory status B. Wond status C. Respiratory status D. Hydration status 506. Prior to administration of an albuterol nebulizer, the nurse should help the patient assume what position? A. Sitting and leaning forward B. Feet elevated above level of heart C. High fowlers D. Standing 507. A patient is presents to the doctor’s office 2 week status post right sided mastectomy. The nurse needs to measure the blood pressure. Which would be the best site? A. Above the left brachial artery B. Right poplitiel artery C. Above the right brachial artery D. Left poiteal artery 508. A child with cystic fibrosis exacerbation presents to the emergency room. Which nursing diagnosis takes First priority in planning for intervention ? A. Imbalanced nutrition related to increased metabolic requirements because of mal absorption B. Defficient knowledge regarding prevention of cystic fibrosis exacerbation C. Impaired gas exchange related to airway obstruction due to mucus D. Interrupted family process 509. During surgery the nurse is assigned the following duties, setting up the sterile field, preparing sutures and ligatures, assisting the surgeon during the procedure by anticipating the instruments and supplies that will be required and labelling tissue specimen obtained during surgery. The nurse is most likely performing what role? A. Circulating nurse B. Scrub nurse C. RN first assistant D. Nurse anesthetic Compiled by : V.Devi | 9176868900

Page 77

Diya Coaching Centre for Nurses One candle wipes out darkness…….

510. A nurse completes discharge instructions for patient who ws admitted 5 days ago with pneumonia. Which statement by the patient would lert the nurse that more discharge teaching is needed? A. Ineed to gradually increase my activities B. I will not need the influenza or pneumonia vaccine C. I my experience the fatigue and weakness for a prolonged time D. I need to have another chest x-ray in 4-6 weeks 511. The nurse is assessing a patient recently diagnosed with AIDS. Which of the following nursing diagnosis has Priority? A. Fear of disease progression, treatment effects, isolation and death related having AIDS. B. Risk of infection related to immunodeficiency. C. Ineffective brething pattern related to opportunistic infections D. Disturbed body image related to rapid body changes from debilitating disease 512. A patient is planning to have an elective surgical procedure to repair an umbilic hernia. The patient is 68 years of old, weight 136 kg(300lbs)and has a diabetis mellitus. Which of the following approaches woud be the most beneficial inorder to reduce the patient’s surgical risk? A. Monitor blood glucose level monthly B. Avoid fluid overload by restricting fluids C. Discourage any changes in routine before surgery D. Encourage weight reduction 513. A nurse caring for a patient following cardiac catheterizartion evaluates the patient’s post procedure, which of the following signs and symptoms would most likely indicate the patient having vagal reaction? A. Diaphoresis B. Chest pain C. Tingling in extremities D. Hematoma formation 514. A home health nurse visits a patient who is newly diagnosed with diabetes. The glucose level ranges from 120mg/dl-150mg/dl, while current Hb AIC level is 6.9%. The patient is complaint with taking prescribed hypnoglycemic medications and eats 3 meals a day followed by desserts sweetened with granulated sugar. The patient also exercise 30 minutes a day 3 times a week. Which of the following education intervention take priority? A. Glucose monitoring B. Medications C. Dietary requirements. D. Exercise regimen 515. Which of the following best describes in the assessment step of the nursing process? A. Identifying nursing interventions as appropriate for short term, immediate and long term goal attaintment B. Assigning priorities to the nursing priorities C. Establishing goals of expected outcomes D. Obtaining a nursing history and complete a physical examination of the patient Compiled by : V.Devi | 9176868900

Page 78

Diya Coaching Centre for Nurses One candle wipes out darkness…….

516. A nurse is measuring the chest tube drainage of a patient who had open heart surgery 4 hours ago. Which of the following is the maximum hourly amount of chest tube drainage that is expected in this time frame? A. 100ml B. 200ml C. 300ml D. 400ml 517. A patient report difficulty sleeping through the night since the death of spouse 6 monts ago. Which of the following is an appropriate long term goal? A. Feeling well rested each morning B. Not feeling tired each afternoon C. Taking brief naps in the middle of the day D. Using a sleeping aid on the nightly basis 518. A patient with systemic lupus errthematous(SLE) reports decreased urinary output during the past 2-4 days and chest pain that is aggrevated by breathing and coughing. The patients vital signs remain within the baseline and S1 and S2 are present with audible friction rub. Which of the following statement would be appropriate for the nurse to make ? A. It sounds like SLE is being well controlled B. It need to get some nytroglycerene for your chest pain C. There may be some inflammation surrounding your heart D. Your symptoms may be due to Urinary tract infections(UTI) 519. A patient has been hospitalised with diagnosis of Chron’s disease. The nurse can best determine the patient’s hydration level by monitoring the: A. Colour of urine B. Brightness of eyes C. Capillary refill in nail bed D. Temperature of lower extremities 520. A patient who had abdominal surgery 6 days ago, has been ambulating the halls with out much difficulty. However on the 7 th post operative the patient complains of increased pain at incisional site and is walking hunched over. The most likely cause of the change is : A. Over ssertion the day before B. Pulmonary edema C. Wound infection D. Deep vein thrombosis 521. A diabetic patient come to the office for follow up 6 weeks after undergoing below the knee amputation of the right leg for gangrene. The nurse observes that he patient is progressing well with the use of a prosthesis and that the skin is in fact. The patient report being generally pain free wnd itching of the right ankle. What should the nurse do/ A. Notify the doctor that there appears to be nerve damage to the right leg B. Refer to a [pain management specialist for long term management C. Refer to Psychiatrist for evaluation since the patient hs no right ankle. D. Explain the phenomena of phantom pain and phantom sensation to the patient Compiled by : V.Devi | 9176868900

Page 79

Diya Coaching Centre for Nurses One candle wipes out darkness…….

522. A 1 year old child presents at the clinic one week after hospitalisation for surgical repair of a fractured right femur. The patient is receiving pain medications every morning and evening. The best way to evaluate the effectiveness of the pain management plan: A. To ask child in simple terms about the comfort level over the past weeks B. By direct observation of the child’s non verbal behaviour during the visit. C. To teach the child how to use the Wong/Baker Faces ain rating scale D. To interview the parent about behaviour moods and sleep pattern over the past weeks 523. A patient visits the clinic for first time. In order to perform an accurate and complete assessment, which of the following would be the nurse’s first step? A. Obtain temperature, pulse and respiration B. Obtain a complete history C. Obtain a blood pressure D. Perform a review of systems 524. The nurse is assigned to care for patients who have recently been diagnosed with Crohns’s disease. The best treatment is usually? A. Dietary changes B. Reversible colostomy C. Permanent colostomy D. Watchful waiting 525. A Patient comes to the medical office with complains of some urinary incontinence. The nurse discovers the incontinence occurs because of an inability delay voiding long enough to reach a toilet after the patient feels a sensation of bladder fullness.This type of incontinence is: A. Stress B, Urge C. Overflow D. Functional 526. When caring for chid with spina-bifuda, the nurse knows that the child has increased risk for allergy to: A. Peanuts B. Straw berrys C. Eggs D. Latex 527. A chid has been admitted with third degree burns covering approximately 8%..... of the body. Which of the following nursing diagnosis takes First priority? A. Imbalance nutrition related to hyper metabolic state B. Disturbed body image related to patients perception of altered appearances. C. Impared skin integrity related to burn injury D. Deficient fluid volume related to fluid shift 528. While planning a class on pregnancy the nurse should include symptoms of pregnancy that must be reported immediately such as: A. Leg cramps B. Visual disturbances C. Swelling of the legs Compiled by : V.Devi | 9176868900

Page 80

Diya Coaching Centre for Nurses One candle wipes out darkness…….

D. Constipations 529. Which of the following reacts to viruses and bacteria by increasing number? A. Antigen B. Antibodies C. Rh factor D. Platelets 530. Which nursing diagnosis takes priority for newly diagnosed patient with a sided stroke? AS. Risk of impaired swallowing related to absent gag reflex B. Risk for impaired skin integrity related to immobility C. Risk for infection related to invasive line placement D. Risk for impaired speech related to left side stroke 531. A nurse taking care of a patient who underwent abdominal surgery 3 years ago. The patient has not been able to breath deeply and refuses to get out of bed since surgery due to pain. Also the patient complaints of shortness of breath and the sounds are diminished upon auscultation. The vital signs are BP; 120/70mm of hg. HR-75, RR-22, T-36.40c (97.6F),> which of the following should the nurse suspected? A. Sepsis B. Atelectasis C. CHF D. Emphysema 532. The nurse assessing a 15 year old patient who is being admitted due to an excacerb ation of bronchial asthma. The nurse should give priority to tasking if the patient has a history of: A. Indoor allergies B. Intubation C. Chest trauma D. Coxsackle virus 533. A community health visits a patient who had a right foot amputation> Which of the following would suggest that the patient is meeting the expected outcome for this type surgery? A. Stays in bed B. Verbalize constant pain C. Avoid social gathering D. Accepts altered body image 534. While reviewing stress management techniques with a patient diagnosed with multiple Sclerosis. What would the nurse identify as Most appropriate ? A. Relaxation in warm bubble bath B. Yoga in a cool rom C. Sun bathing D. Cross country running 535. A nurse visits the home of a patient who is one week post left breast mastectomy. Which of the following should be included in patient education? A. It is OK to use a straight edge razor when shaving B. Blood pressure checks should be done in the left arm C. Cuticle should not be cut Compiled by : V.Devi | 9176868900

Page 81

Diya Coaching Centre for Nurses One candle wipes out darkness…….

D. Avoid insect repellent on the left arm 536. A Patient is 24 hours post operative after having a right total hip arthroplasty. The patient complaints of the pain in the right calf rated 6 on a scale 0 (no pain) to 10(severe). The nurse observes that the right calf is warm and tender to touch, which recognizes that they are classical sign of : A. Ineffective tissue perfusion B. Fluid overload C. Arterial occlusion D. Deep vein thrombosis 537. A patient with dementia treated for dehydration. The patient is confused and has been immobile for the past month. Currently the patient is incontinent and unable to feed self. The nursing care plan would include: A. Coughing and deep breathing exercises B. Positioning and turning every 2 hours C. Range of emotion exercises to all extremities every hour D.Ambutate at least 20 steps every shift 538. A chid comes in to the clinic with severa lesions on the scalp. The rund lesions have dandruff like scaling with hair loss. What is the most likely diagnosis for this skin condition? A. Impetigo B. Ring worm (tinea capitis) C. Pediculosis capitis D. Scabies 539. A nurse evaluate a patient for signs of rebleeding from ruptured intracranial aneurysm that required surgical ligation. The highest risk for aneurysm rebleed is within? A. 6 hours B. 24 hours C. 48 hours D. 72 hours 540. When discussing dietary choices of a patient who is receiving heparin therapy, the nurse should stress that which of the following foods affect the cloting time? A. High protein diet B. Soy-based foods C. Food high in Vitamin K D. Foods containing Cows milk. 541. A patient admitted to the hospital for pneumonia, finishes a course of levofloxacin(L vaquin) lungs are clear and the patient is no longer coughing. Which of the following post discharge laboratory results best indicates desired outcomes? A. Normal WBC B. Normal haematocrit count C. Normal platelet count D. Normal potassium

Compiled by : V.Devi | 9176868900

Page 82

Diya Coaching Centre for Nurses One candle wipes out darkness…….

542. A home health care nurse visits a patient diagnosed with rheumatic. The nurse gathers information about the pain level after the use of the prescribed pain medication to check the effectiveness of the intervention. This phase of nursing process is called? A. Assessment B. Diagnosis C. Implementation D. Evaluation 543. A plan of care for a child with cerebral palsy should include all of the following Expect? A. Physical therapy B. Play C. Feeding D. Bowel and bladder training 544. A patient is admitted to the hospital with Klebsiella pneumonia. During the initial intravenous dose of amikin(amikacin sulphate) the patient develop severe respiratory distress. This is most likely? A. A side effect B. An indication of drug tolerance C. A drug allergy D. A toxic effect 545. A patient is diagnosed with peptic ulcer> What would be the long term goal for this patient? A. Patient remains free from the signs and symptoms of gastro intestinal bleeding B. Patient maintains ife style alterations to prevent recurrence of ulcer. C. Patient expresses decreased pain level D. Patient performs activities of daily living without difficulty 546. A patient experienced cerebrovascular accident (CVA) one week ago. The doctor orders the patient discharged from the hospital to home health care. The visiting nurse evaluates the patient and notes, the patient has difficulty initiating speech and forming words. Which of the following type of aphasia is most likely is this? A. Receptive B. Expressive C. Fluent D. Conduction 547. A Nurse is evaluating the home of patient with left sided paralysis. Which of the following observation would indicate that the patient is complying with home-based safety? A. The telephone is on the bed side table which is next to the head of the bed. B. The bedside commode is on the left side of the bed with back of the commode facing the foot of the bed. C. The walker has wheels on its back legs and has tennis balls on the front legs D. The stair leading from the bedroom to the living area a handrail on the right side of the stairway. 548. A patient is admitted to the hospital with a CVA; right hemipegia and expressive aphasia. With a nursing diagnosis of impared verbal communication, what is the Best short term goal for this patient? Compiled by : V.Devi | 9176868900

Page 83

Diya Coaching Centre for Nurses One candle wipes out darkness…….

A. Learn to speak clearly in 30 days B. Communicate effectively within one week C. Have al needs anticipated by staff daily D. Make basic needs known daily 549. A patient has a dissection aneurysm. The patient’s surgery would be categorized as: A. Elective B. Urgent C. Emergency D. Diagnostic 550. A parents to the emergency room due to overdose of morphine sulphate. Which of the following should the nurse have readily available? A. Glucogen B. Antibiotic C. Acetycysteine D. Nalaxone (narcan) 551. A patient with iron deficiency anaemia due to an insufficient iron intake need to learn to select better food choices. The nurse works with this patient to establish a plan of care and provide education on proper nutrition and good source of iron. Besides educating the patient on a well balanced diet the nurse would Most likely teach the patient that good sources of iron include: A. Seafood, cheese, soya bean oil and chocolate B. Animal protein, egg yolk ,dried fruits and nuts C. Dairy product, citrus fruits, fish liver oils and poultry products D. Sea food, fruit, poultry and tomatoes 552. A 42 year old patient is in lower body cast following a motor vehicle accident. In order to minimise muscle strength loss while in the cast, the nurse will instruct the patient in the performance of A. Isometric exercises B. Passive range of motion exercises C. Active range of motion exercises D. Resistive range of motion exercises 553. A patient is being followed in the clinic for hypertension, adult onset diabetes and obesity. The patient is apathetic about learning nutritional guidelines to reach the goals of weight loss and consumption of a healthy diet. The patient admitted to eating “whatever is put in front of me”. Which of the following actions would the nurse take? A. Colaborrate with the patient to set goals B. Add a nursing diagnosis of non compliance C. Refer for psychiatric screening for depression D. Discuss nutritional intervention with the spouse 554. A child is admitted to the paediatric ward with fever, leathery joint pain and abdominal pain for several weeks. The patient has a history of recurrent respiratory and ear infections. Physical findings include widespread ecchymosis, generalized lymphadenopathy, hepatomegaly and pallor. Lab works show a low haemoglobin level, low RBC evel, low haematocrit and platelets. The nurse should expect the bone marrow stain to show? Compiled by : V.Devi | 9176868900

Page 84

Diya Coaching Centre for Nurses One candle wipes out darkness…….

A. large number of lymphoplasts and lymphocytes B. Low number of lymphoplasts and large number of lymphocytes C. Low number of lymphopasts and lymphocytes D. Large number of lymphoplasts and low number of lymphocytes 555. Immediately following the birth of a full term newborn, which of the following nursing diagnosis should tke priority? A. Ineffective airway clearance related to nasal and oral secretions B. Ineffective thermoregulation related to environmental factors C. Risk for imbalanced fluid volume related to weak sucking reflex D. Risk for injury related to immature defence mechanism 556. A patient receives intravenous therapy of 1000cc normal saline with 20 MEq potassium chloride at a rate of 75cc per hour. Upon evaluation of the site there is no edema, the vein appears slightly red and the patient complaints of pain. Wht should the nurse do/ A. Slow the rate to prevent burning from the solution and continue to monitor B. Discontinue the intravenous line and restart in another site C. Monitor at least every half hour for edema but continue as order state. D. Notify the doctor that the patient is having an adverse reaction to themediction 557.A healthy 26 year old patient is at 39 weeks gestation. The patient is not considered high risk at the time of admission to the labour and delivery unit. Which of the following pending laboratory test results should receive priority? A. RBC count B. Haematocrit C. WBC count D. Blood type 558. A patient comes to the emergency department with extreme dyspnea, orthopnea anxiety and complaints of feeling panicky. The patient is coughing up frothy sputum and is cyanotic with profuse perspiration. Inspiratory and expiratory wheezing and hubbling sounds are auscultated. The patient is diagnosed with acute pulmonary edema. What should the nurse do first? A. Identify precipitating factors and underlying conditions B. Administering morphein (Duramorph) to reduce anxiety C. Assess oxygen saturation rate. D. Administer digoxin (lanoxin) to decrese fluid backing up into the lungs 559. During surgery the patient has the following intake and output; Intravenous fluids -650cc, intravenous antibiotic-50cc; 1 Unit of red blood cels (PRBC)-350cc; Nsogastric output-120cc, Estimated blood loss 80cc. And urine in the foley’s catheter is 240cc. What is the patients total output? A. 120cc B. 200cc C. 240cc D. 440cc 560. A 25 year old female presents to the emergency department room with leathary, decreased reflexes, hypoventilation, hypotension and fixed dilated pupils. A family member who is accompanying the patient has an empty bottle of diazepam (valium) which the label states was Compiled by : V.Devi | 9176868900

Page 85

Diya Coaching Centre for Nurses One candle wipes out darkness…….

recently refilled. The family member also indicates that the patient has a history of depression. What intervention should the nurse expect to administer? A. Nalaxone (Narcan) B. Activated charcoal C. Magnecium sulphate to reduce the risk of seizure D. Tap water enema 561. A nurse is signed to acre for patient with a diagnosis pf thrombotic stroke. The nurse knows that this type of stroke is most likely caused by? A. Blockage of large vessels as a result of artherosclerosis B. Emboli produced from vascular heart disease C. Decreased cerebral blood flow due to circulatory failure D. A temporary disruption of oxygenation of the brain 562. The nurse administered a prescribed intra muscular medication to a patient during a home visit. How should the nurse dispose the used needle and syringe? A. Recap the needle, then place the needle and syringe into a aterproof continer until sfe disposal can be made. B. Bend the needle back towards the barrel of the syringe before putting the needle and syringe in a met trash container C. wrap the needle and syringe in disposable paper before putting the needle and syringe into the dirty section of the nurses equipment bag D. Put the needle and syringe directly into a puncture resistant plastic container that has a lid 563. What is the most common characteristic of stage 4 pressure ulcer? A. Pink skin B. Presence of sinus tracts C. Exposure of bones D. Infection 564. While visiting a patient with a new colostomy, the home care nurse observes that the skin around the stoma cite is red. Which inventions should the nurse do next? A. Apply pectin, gelatine, or synthetic skin barrier around the stoma. B. Apply triple antibiotic to the raw skin and leave it open to the air C. Instruct to empty the pouch as soon as stool is present D. Instruct to remove the bag and skin barrier after each stool 565. A nurse educates a patient about the use of incentive spiro metry to prevent atelectasis after a surgery. The nurse is performing what step of nursing process? A. Diagnosis B. Assessment C. Implementation D. Evaluation 566. A patient had suicidal tendency. He was very depressed and inactive. Doctor prescribed SSRI and he took this medicine around 1 month. Now he is very active and energetic but suicidal tendency is still with him. What is his grade of suicidal tendency? A, None Compiled by : V.Devi | 9176868900

Page 86

Diya Coaching Centre for Nurses One candle wipes out darkness…….

B. Mild C. Moderate D. Severe 567. A patient likes to tke enema himself. Which position is most suitable? A. Sims B. Left lateral C. Trendelenberg D. Prone 568. Cardiac surgery position? A. Trendelenberg B. Supine C. Dorsal recumbent D. Semi fowler’s 569. A patient is on heparin therapy and the patient continues to bleed during therapy. Which medication should be administered to prevent it? A. Morphine B. Vit.K C. Nalanone D. Flumazenil 570. What is the priority action of a nurse for a patient with following vitas emergency room? Pulse -86/mt Respiratory rate -22/mt Blood pressure -100/60 mm of hg Temperature -390C A. Normalize the temperature B. Continue to monitor vitals C. Inform the doctor D. Document the event 571. A patient comes to the emergency department with cyanosis and edema. Which position to be given? A. Trendelenburg B. Head elevated position C. Semi flower position D. Sims position 572. A patient has undergone surgical removal of parathyroid gland. After 2 weeks the patient left a tingling sensation, numbness in the extremities. The nurse should prescribe which type of food products? A. Cheese, orange, meat B. Whole milk, Grains, fish C. Green leafy vegetables D. Fruit juices

Compiled by : V.Devi | 9176868900

Page 87

Diya Coaching Centre for Nurses One candle wipes out darkness…….

573. A patient has tenderness on the right upper quadrant and pain on the abdomen. Abdominal distention is present on prescription dull sound can be heard. On the patients right side also bluish discolouration on the umbilical region. Which may be the condition? A. Hepatomegly B. Hiatal hernia C. Gstric ulcer D. Ascitis 574. For a patient having seizure, which management goes to priority? A. Side lateral position B. O2 administration C. Restrain the patient D. Use mouth pieces 575. A patient is on heparin treatment started warfarin. For this patient which lab test should be done? A. APTT B. Bilirubin level C. PTT INR D. Ph of the blood 576. A chid with type1 Diabetes mellitus has taken morning dose insulin and went for sport activities. The child suddenly became cool and confused. What is the immediate management? A. Provide orange juice 1 glass B. 5% dent rose bolus administration C. Provides 2 mg glucagon D. Provide candy to the child 577. A child with attention deficit has brought to a primary health nurse. The teachers tod that the child is very restless and not attentive in the class. The best suggestion by the nurse is A. Make the child to follow strict schedule B. Make him to sit with a grade children C. Make him to set at last bench D. Place the child in a less stimulated class 578. A patient fall from the bed. The nurse makes him to sit on the bed. The patient has rapid swallow breathing, shortening of limb, profuse sweating and pain on the leg. Which management gets the priority? A. Pain management B. O2 administration C. Immobilisation of affected leg D. Check V/s 579. Precaution for ‘Diphtheria’ A. Droplet B. Airborne C. Contact D. Reverse isolation Compiled by : V.Devi | 9176868900

Page 88

Diya Coaching Centre for Nurses One candle wipes out darkness…….

580. An obese patient, who is not interested to mobilize properly, has calf muscle swelling , pain and cannot even raise the leg. Which drug can be infused to treat this condition? A. heparin injection B. Hot, warm application on the leg C. Compression stockings D. Immobilize the leg 581. A post op patient requested for pain relief, the nurse has administered morphiene 5 mg according to the order. After 10 minutes the patient become confused decreased respiration rate and with no response. BP 100/70mm of Hg HR 110/mt RR 8/mt SPO2 88 % Temp 36.40C What is the priority management? A. Administer Oxygen B. Barium enema C. Administer nalaxone D. Administer IV fluids 582. A child came with the complaints of nausea, vomiting, bloody stool and mucus. Which laboratory test should be assessed? A. Hb count B. Barium enema C. Urine list D. Fecal occult blood test 583. A patient came to the hospital with a abdominal pain and nausea. Diagnosed on having stone in the bile duct. Which assessment can consume this? A. Dark and tarry stool B. Amber colour urine C. Yellowish sclera D. Edema on Extremities

Compiled by : V.Devi | 9176868900

Page 89

Related Documents

Questions
January 2021 1
Average Questions
January 2021 0
Refresher Questions
February 2021 1
Law Questions
January 2021 1
Proe Questions
February 2021 1
Trigonometry Questions
February 2021 3

More Documents from "Randy Viola"